full length 7

71
MCAT Full-Length Tests Dear Future Doctor, The following Full-Length Test and explanations are an opportunity to bring it all together in simulation. Do not engage in Full-Length practice until you have adequately prepared your knowledge and critical thinking skills in Subject, Topical, and Section tests. Simply g the tests is inadequate; a solid understanding of your performance through your Score Reports and the explanations is necessary to diagnose your specific weaknesses and address them before Test Day. All rights are reserved pursuant to the copyright laws and the contract clause in your enrollment agreement and as printed below. Misdemeanor and felony infractions can severely limit your ability to be accepted to a medical program and a conviction can result in the removal of a medical license. We offer this material for your practice in your own home as a courtesy and privilege. Practice today so that you can perform on test day; this material was designed to give you every advantage on the MCAT and we wish you the best of luck in your preparation. Sincerely, Albert Chen Executive Director, Pre-Health Research and Development Kaplan Test Prep © 2003 Kaplan, Inc. All rights reserved. No part of this book may be reproduced in any form, by Photostat, microfilm, xerography or any other means, or incorporated into any information retrieval system, electronic or mechanical without the written permission of Kaplan, Inc. This book may not be duplicated, distributed or resold, pursuant to the terms of your Kaplan Enrollment Agreement.

Upload: lucero-cas

Post on 02-Jun-2015

343 views

Category:

Health & Medicine


1 download

DESCRIPTION

MCAT Practice

TRANSCRIPT

Page 1: Full length 7

MCAT Full-Length Tests

Dear Future Doctor, The following Full-Length Test and explanations are an opportunity to bring it all together in simulation. Do not engage in Full-Length practice until you have adequately prepared your knowledge and critical thinking skills in Subject, Topical, and Section tests. Simply g the tests is inadequate; a solid understanding of your performance through your Score Reports and the explanations is necessary to diagnose your specific weaknesses and address them before Test Day. All rights are reserved pursuant to the copyright laws and the contract clause in your enrollment agreement and as printed below. Misdemeanor and felony infractions can severely limit your ability to be accepted to a medical program and a conviction can result in the removal of a medical license. We offer this material for your practice in your own home as a courtesy and privilege. Practice today so that you can perform on test day; this material was designed to give you every advantage on the MCAT and we wish you the best of luck in your preparation. Sincerely,

Albert Chen Executive Director, Pre-Health Research and Development Kaplan Test Prep © 2003 Kaplan, Inc. All rights reserved. No part of this book may be reproduced in any form, by Photostat, microfilm, xerography or any other means, or incorporated into any information retrieval system, electronic or mechanical without the written permission of Kaplan, Inc. This book may not be duplicated, distributed or resold, pursuant to the terms of your Kaplan Enrollment Agreement.

Page 2: Full length 7

Physical SciencesTime: 100 Minutes

Questions 1–77

DO NOT BEGIN THIS SECTION UNTIL YOU ARE TOLD TO DO SO.

Page 3: Full length 7

2

GO ON TO THE NEXT PAGE.

PHYSICAL SCIENCES

DIRECTIONS: Most of the questions in the PhysicalSciences test are organized into groups, with adescriptive passage preceding each group of ques-tions. Study the passage, then select the single bestanswer to each question in the group. Some of thequestions are not based on a descriptive passage; youmust also select the best answer to these questions. Ifyou are unsure of the best answer, eliminate thechoices that you know are incorrect, then select ananswer from the choices that remain. Indicate yourselection by blackening the corresponding circle onyour answer sheet. A periodic table is provided belowfor your use with the questions.

1

H

1.0

2

He

4.0

3

Li

6.9

4

Be

9.0

5

B

10.8

6

C

12.0

7

N

14.0

8

O

16.0

9

F

19.0

10

Ne

20.2

11

Na

23.0

12

Mg

24.3

13

Al

27.0

14

Si

28.1

15

P

31.0

16

S

32.1

17

Cl

35.5

18

Ar

39.9

19

K

39.1

20

Ca

40.1

21

Sc

45.0

22

Ti

47.9

23

V

50.9

24

Cr

52.0

25

Mn

54.9

26

Fe

55.8

27

Co

58.9

28

Ni

58.7

29

Cu

63.5

30

Zn

65.4

31

Ga

69.7

32

Ge

72.6

33

As

74.9

34

Se

79.0

35

Br

79.9

36

Kr

83.8

37

Rb

85.5

38

Sr

87.6

39

Y

88.9

40

Zr

91.2

41

Nb

92.9

42

Mo

95.9

43

Tc

(98)

44

Ru

101.1

45

Rh

102.9

46

Pd

106.4

47

Ag

107.9

48

Cd

112.4

49

In

114.8

50

Sn

118.7

51

Sb

121.8

52

Te

127.6

53

I

126.9

54

Xe

131.3

55

Cs

132.9

56

Ba

137.3

57

La *

138.9

72

Hf

178.5

73

Ta

180.9

74

W

183.9

75

Re

186.2

76

Os

190.2

77

Ir

192.2

78

Pt

195.1

79

Au

197.0

80

Hg

200.6

81

Tl

204.4

82

Pb

207.2

83

Bi

209.0

84

Po

(209)

85

At

(210)

86

Rn

(222)

87

Fr

(223)

88

Ra

226.0

89

Ac †

227.0

104

Rf

(261)

105

Ha

(262)

106

Unh

(263)

107

Uns

(262)

108

Uno

(265)

109

Une

(267)

*

58

Ce

140.1

59

Pr

140.9

60

Nd

144.2

61

Pm

(145)

62

Sm

150.4

63

Eu

152.0

64

Gd

157.3

65

Tb

158.9

66

Dy

162.5

67

Ho

164.9

68

Er

167.3

69

Tm

168.9

70

Yb

173.0

71

Lu

175.0

90

Th

232.0

91

Pa

(231)

92

U

238.0

93

Np

(237)

94

Pu

(244)

95

Am

(243)

96

Cm

(247)

97

Bk

(247)

98

Cf

(251)

99

Es

(252)

100

Fm

(257)

101

Md

(258)

102

No

(259)

103

Lr

(260)

PERIODIC TABLE OF THE ELEMENTS

Page 4: Full length 7

Passage I (Questions 1–6)

In 1665, Isaac Newton presented his law of universalgravitation, which states that every body in the universeattracts every other body with a force that is proportionalto the product of the masses and inversely proportional tothe square of the intervening distances. The strength ofthese gravitational forces depends on a constant of propor-tionality known as the gravitational constant, G. In 1798,Henry Cavendish performed an experiment to measure thevalue of G. Figure 1 shows a schematic of the Cavendishapparatus, which consists of two large, identical sphericalbodies of mass M, and two small, identical spherical bod-ies of mass m that are attached to the ends of a rod oflength L. A thin quartz fiber is used to suspend the rod.The suspended rod and two small bodies form a torsionbalance that is free to rotate about the quartz fiber.

Figure 1

When the rod and two small bodies are suspended fromthe end of the quartz fiber, each small body experiences anattraction for each neighboring large body. These gravita-tional forces exert a torque on the torsion balance, causingit to rotate as the distance between the small and large bod-ies decreases. While the torsion balance turns, the quartzfiber becomes twisted. As a result, the quartz fiber exerts acounter-torque, acting in opposition to the gravitationaltorque caused by the attraction between the bodies. Thefarther the torsion balance turns, the greater the counter-torque exerted by the quartz fiber. When the torque exertedby the quartz fiber equals the gravitational torque, the tor-sion balance comes to rest at an equilibrium position. Withknowledge of the torque exerted by the quartz fiber and thedistance between the large and small bodies, the value ofthe gravitational constant G can be determined.

1. In the experiment, the torsion balance is suspendedfrom the end of the quartz fiber and is held fixed.Neglecting the Earth’s gravitation, the energy of thesmall bodies is primarily:

A. kinetic energy.

B. gravitational potential energy due to the pres-ence of the two large bodies.

C. potential energy due to torsion of the quartzfiber.

D. rotational kinetic energy of the quartz fiber.

2. When the torsion balance is first released, it rotates asthe small bodies move slowly toward the large bod-ies. Neglecting air resistance and the torque exertedby the quartz fiber, the small bodies are:

A. accelerating.

B. moving with constant velocity.

C. decelerating.

D. stationary.

3. While the small bodies are moving toward the largebodies, which one of the following takes place?

A. Kinetic energy is transformed into gravitationalpotential energy.

B. Gravitational potential energy is transformedinto kinetic energy.

C. Torsional potential energy of the quartz fiber istransformed into kinetic energy.

D. Torsional potential energy of the quartz fiber istransformed into gravitational potential energy.

4. Once the small bodies have come to rest, it is foundthat both of them are located a distance R away fromeach neighboring large body. Based on this, whichone of the following correctly expresses the torqueacting on the torsion balance at that instant?

A. τ =

B. τ =

C. τ =

D. τ = GMmL�

R2

GMmL�

2R2

GMm�

2R2

GMm�

R2

quartz fiber

M

m

M

L

m

3

GO ON TO THE NEXT PAGE.

Page 5: Full length 7

5. Which one of the following modifications will makethe Cavendish experiment more sensitive?

A. Decreasing the length of the rod

B. Decreasing the mass of the small bodies

C. Increasing the thickness of the quartz fiber

D. Increasing the mass of the large bodies

6. If the gravitational constant G is measured to be 6.7 × 10–11 N·m2/kg2, and smaller spheres of mass3 × 10–6 kg are attached to a 2.5-meter rod, then whatis the gravitational force between the two smallerspheres?

A. 3.2 × 10–17 N

B. 8.0 × 10–17 N

C. 9.6 × 10–23 N

D. 2.4 × 10–22 N

Passage II (Questions 7–12)

The hypothetical compounds known as ideal gasesoften give chemists great insight into the behavior of realgases. The ideal gas approximation arises from basictrends deduced in the behavior of real gases at low pres-sure. Boyle’s law, for instance, results directly from theassumptions underlying the kinetic theory of gases:

1. A pure gas consists of a large quantity of identicalmolecules separated by very large distances relativeto the size of the molecules.

2. Molecules move in random directions with a distribu-tion of speeds.

3. Molecules exert no forces on one another betweencollisions.

4. Collisions of molecules are elastic; no energy is lost.

Though the ideal gas approximation is accurate nearatmospheric pressure, at high densities the behavior of realgases diverges from the ideal model, since these conditionsinvalidate one or more of the kinetic theory’s assumptions.To account for these deviations, the nineteenth-centuryDutch physicist Johannes van der Waals proposed appro-priate corrections to the ideal gas law, expressed by the vander Waals equation of state, Equation 1.

(P + )(V – nb) = nRT

Equation 1

The parameters a and b are functions of the physicalcharacteristics of specific molecules. The van der Waalsconstants for N2 and H2 are listed below in Table 1.

Table 1 Van der Waals Constants for N2 and H2

When deciding whether to treat an unknown gas asideal, chemists must determine how accurate they requiretheir results to be, and include multiple factors in theirdecision: ambient pressure, intermolecular attractions, andthe size of gas molecules.

One such chemist attempts to analyze an unknowncompound twice. The chemist assumes the gas is ideal inhis first analysis and real in his second. To gauge the accu-racy of his first assumption, the chemist calculates thecompressibility ratio, Z, as a function of pressure, Z =PV/nRT.

Van der Waals Constants:a b

(atm L–1 mol–2) (L mol–1)

Nitrogen: 1.390 0.03913Hydrogen: 0.2444 0.02661

n2a�V2

4

GO ON TO THE NEXT PAGE.

Page 6: Full length 7

7. A chemist’s assumption that an unknown gas is idealwould be less valid if molecules of the compoundwere found to be:

A. non-polarizable.

B. rich in helium.

C. large.

D. in continuous, random motion.

8. As temperature increases, the volume of 3 moles ofthe unknown gaseous compound will:

A. decrease if the compound is ideal, increase if thecompound is real.

B. decrease if the compound is ideal, decrease ifthe compound is real.

C. increase if the compound is ideal, increase if thecompound is real.

D. increase if the compound is ideal, decrease if thecompound is real.

9. The chemist discovers that the unknown compoundhas a molecular weight of 46.01 g/mol. What is thespecific gravity of the compound at standard temper-ature and pressure?

A. 2.06

B. 0.485

C. 9.20 × 10–2

D. 2.06 × 10–3

10. If a is the van der Waals constant found in the equa-tion of state, rank the a values of the followinggaseous compounds:

I. water

II. hydrogen

III. carbon dioxide

A. I > III > II

B. II > III > I

C. III > I > II

D. III > II > I

11. It is expected that as the attractive forces on amolecule increase, the compressibility ratio will:

A. increase linearly.

B. decrease linearly.

C. increase proportional to 1/a.

D. remain constant.

12. Under which set of conditions can the volume ofH2O (g) best be approximated by nRT/P?

A. 1.00 atm, 100°C

B. 500 atm, 100°C

C. 1.00 atm, 150°C

D. 500 atm, 150°C

5

GO ON TO THE NEXT PAGE.

Page 7: Full length 7

Passage III (Questions 13–18)

Engineers have devised a novel system of rapid trans-portation between widely separated places on Earth. If asmooth, frictionless tunnel is dug between two cities, anda capsule is placed just inside the tunnel, then gravity willdraw the capsule into the Earth, and deliver it to the stationat the other end of the tunnel. Figure 1 shows what thetransportation system looks like between two cities at thesame latitude θ.

Figure 1

As the capsule is drawn into the Earth, the gravitationalforce acting on it decreases. Because the Earth encloses thecapsule, only the fraction of the Earth’s mass that liescloser to the center of the Earth than the capsule exerts anet force on the capsule. So when the capsule is at a dis-tance r < RE from the center of the Earth, the componentof the force on the capsule in the direction of the center of

the tunnel is Ftunnel = ( )x, where x is the distance

to the center of the tunnel. This equation is a form ofHooke’s Law: since the force causing the capsule to accelerate is directly proportional to the distance from the center of the tunnel, the capsule undergoes simple harmonic oscillation. The amplitude of oscillation Adepends on the latitude of the tunnel, A = RE cosθ. (Note:Assume air resistance is negligible. The gravitational constant G = 6.37 × 10–11 N·m2/kg2, the radius of theEarth RE = 6.37 × 106 m, and the mass of the Earth ME = 5.98 × 1024 kg.)

13. What is the period of oscillation of a 1000 kg cap-sule?

A. T = 2π��B. T = 2π��C. T = ��D. T = ��

14. How does the maximum velocity of the capsule varywith latitude?

A. The velocity of the capsule is constant.

B. The maximum velocity of the capsule increasesas latitude increases.

C. The maximum velocity of the capsule increasesas latitude decreases.

D. The maximum velocity of the capsule doesn’tdepend on the latitude.

15. In an emergency, the capsule can be stopped in mid-passage. If a 1000 kg capsule is stopped four millionmeters into a trip across a six million-meter tunnel,how much energy will need to be supplied to the cap-sule for it to complete its journey to the other side ofthe tunnel?

A. 6.1 × 109 J

B. 1.2 × 1010 J

C. 4.8 × 1023 J

D. 9.6 × 1024 J

16. What is the acceleration due to gravity of an objecton the surface of Mars?

(RM = 3.37 × 106 m, MM = 6.42 × 1023 kg)

A. 1.85 m/s2

B. 3.70 m/s2

C. 9.81 m/s2

D. 1.4 × 107 m/s2

GME (1000 kg)��

R3E

GME�R3

E

R3E��

GME (1000 kg)

R3E�

GME

GmME�

R3E

Ax

RE

θ

6

GO ON TO THE NEXT PAGE.

Page 8: Full length 7

17. In a tunnel of a given latitude, where does the capsuleundergo maximum acceleration?

A. At the center of the tunnel

B. At the endpoints of the tunnel

C. At the position of maximum speed

D. The acceleration of the capsule is constant.

18. In moving the capsule from one side of a tunnel tothe other, if the tunnel is at latitude θ, how muchwork is done by the Earth?

A. The Earth does no net work on the capsule.

B.

C.

D.2GmME cos2 �

R2E

2GmME cos �

R2E

GmME cos �

R2E

7

GO ON TO THE NEXT PAGE.

Page 9: Full length 7

Passage IV (Questions 19–23)

A relatively unstable compound, hydrogen peroxidewill dissociate through several different pathways. In solu-tion with aqueous bromine, the compound decomposesthrough the two-step method indicated in Equation 1.

Step 1: Br2(aq) + H2O2(aq) → 2Br–(aq) + 2H+(aq) + O2(g)Step 2: 2Br– (aq) + H2O2(aq) + 2H+(aq) → Br2(aq) + 2H2O(l)

Equation 1

A chemist investigates the mechanism of bromine-facilitated decomposition of hydrogen peroxide.

Experiment 1The chemist prepares 250 mL of 0.2 M H2O2(aq) and

at one-minute intervals measures the volume of O2(g)evolved at 1 atm. Three minutes into the experiment, thechemist adds 1 mL of 0.1 M BR2(aq) to the solution andcontinues to take volume measurements. His data is shownin Figure 1.

Figure 1 Volumetric Analysis of H2O2 Decomposition

Experiment 2The chemist repeats the Experiment 1 protocol in two

reaction flasks, A and B. He adds 1 mL and 2 mL, respec-tively, of 0.1 M KI(aq) to flasks A and B. Upon volumet-ric analysis, he discovers that the reaction rate immediatelyfollowing bromine addition is twice as large in flask B.

19. During the decomposition of liquid-phase hydrogenperoxide, bromine behaves as everything EXCEPT:

A. a heterogeneous catalyst.

B. an oxidizing agent.

C. a reducing agent.

D. a weak base.

20. In Steps 1 and 2, respectively, the changes in oxida-tion number of oxygen are:

A. +2, 0.

B. +1, –1.

C. +1, 0.

D. +2, –1.

21. Consider the following mechanism of the oxidationof bromide ions by hydrogen peroxide in acid solu-tion:

H+ + H2O2 → H2O–OH+ (fast)

H2O–OH+ + Br– → HOBr + H2O (slow)

HOBr + H+ + Br– → Br2 + H2O (fast)

Which one of the following rate laws is consistentwith this mechanism?

A. Rate = k[Br– ][H+][H2O2]

B. Rate = k[H2O–OH+][Br–]

C. Rate = k[H2O2][Br–]2[H+]2

D. Rate = k[H2O2][H+]

22. Hydrogen peroxide decomposes slowly in the pres-ence of light. With a measured rate constant,k = 1.06 × 10–3 min–1, how long will it take 1.0 MH2O2(aq) to decompose into 0.25 M H2O2(aq)?

A. × 104 min

B. × 103 min

C. × 103 min

D. × 103 minln 2�1.06

ln 2�2.12

(ln 2)2�

1.06

ln 2�5.3

00 3 6

Time (minutes)

Vol

ume

O2

(mL

)

9 12

250

500

8

GO ON TO THE NEXT PAGE.

Page 10: Full length 7

23. In another experiment, the chemist accurately mea-sures the reaction enthalpy for the decomposition ofhydrogen peroxide. In conjunction to this result, whatthermodynamic value would allow him to calculatethe heat of formation for hydrogen peroxide?

A. The bond dissociation energy of hydrogen

B. The heat of formation for water

C. The bond dissociation energy of bromine

D. The heat of vaporization for water

24. A rider traveling at 9 m/s brings her bicycle to asmooth stop over a distance of 5 meters. What is themagnitude of her acceleration during this time?

A. 1.8 m/s2

B. 8.1 m/s2

C. 9.0 m/s2

D. 16.2 m/s2

25. Using the given table of average bond enthalpies, cal-culate the heat of reaction for the combustion ofpropane:

C3H8(g) + 5O2(g) → 3CO2(g) + 4H2O(g)

A. –3834 kJ/mol

B. –2217 kJ/mol

C. +2217 kJ/mol

D. +3834 kJ/mol

26. A laser emits photons with wavelengths of 413 nm.These photons can cause an atomic electron transi-tion between which of the following atomic energystates?

(Planck’s constant h = 4.2 × 10–15 eV·s; the speed oflight c = 3 × 108 m/s).

A. E1 = –10.4 eV; E2 = –7.4 eV

B. E1 = –9.8 eV; E2 = –7.8 eV

C. E1 = –13.6 eV; E2 = –3.4 eV

D. E1 = –7.4 eV; E2 = –6.4 eV

Bond Average bond

enthalpy (kJ/mol)

C-H 414

C-C 347

C-O 351

C=O 803

O=O 498

O-O 138

O-H 464

9

GO ON TO THE NEXT PAGE.

Questions 24 through 27 are NOT based on adescriptive passage.

Page 11: Full length 7

27. What is the percent ionization of a 0.339 M HCNsolution with pH = 4.89?

A. 3.8 × 10–5%

B. 3.8 × 10–3%

C. 4.4 × 10–6%

D. 4.4 × 10–4%

Passage V (Questions 28–32)

When a capillary tube is inserted into a beaker filledwith pure water, the water rises up inside the tube, asshown in Figure 1. This happens because the attractiveforces between the water and glass molecules are greaterthan the mutual forces between the water molecules withinthe liquid. In general, the surface tension is a property ofthe liquid and the material making up the tube; it is inde-pendent of the tube’s dimensions.

Figure 1

The surface tension of a liquid in a capillary tube actsalong the surface of the liquid, where the liquid is in con-tact with the tube. This force acts upwards, counteractingthe weight of the enclosed fluid, causing the meniscusshown in Figure 2.

Figure 2

The surface tension T of a liquid in a capillary tube canbe determined by measuring the height h of the fluid col-umn, and using the formula where T = 0.5ρghr, where ρ isthe density of the fluid, and r is the inner radius of the tube.

T T

Beaker h

Capillary Tube

Liquid

10

GO ON TO THE NEXT PAGE.

Page 12: Full length 7

28. Assume that the total force due to surface tensionacts entirely along the boundary line between thewater and the glass, as shown in Figure 2. What canwe infer about the force due to surface tension?

A. The force due to surface tension is greater thanthe weight of the water in the tube.

B. The force due to surface tension is equal to theweight of the water in the tube.

C. The force due to surface tension is weaker thanthe weight of the water in the tube.

D. There is not enough information to determinethe relationship between the force due to surfacetension and the weight of the water in the tube.

29. If the inner diameter of the tube is doubled, whateffect can we expect this change to have on the heightof the water in the tube?

A. The new height will be twice as high as before.

B. The new height will be as 3/4 high as before.

C. The new height will be 1/2 as high as before.

D. The new height will be the same as before.

30. If a capillary tube is filled with mercury (ρHg = 1.36 × 104 kg/m3), and an iron sphere (ρFe = 7.86 × 103 kg/m3) is placed on the surface,approximately what fraction of the volume of thesphere remains above the fluid?

A. 8%

B. 42%

C. 58%

D. 79%

31. An experiment performed with a particular liquidshows that the liquid doesn’t go up or down in thetube, but stays at the same level. On the basis of thisobservation, what can we conclude about the inter-molecular forces within the liquid?

A. They are greater than the forces between the liq-uid and glass molecules.

B. They are weaker than the forces between the liq-uid and glass molecules.

C. They are the same strength as the forcesbetween the liquid and glass molecules.

D. There are no intermolecular forces between themolecules in the liquid.

32. If 4.7 cubic centimeters of water (ρ = 1000 kg/m3)are drawn into a tube with a radius of 10 cm, what isthe surface tension?

A. 7.3 × 10–2 N/m

B. 7.3 × 102 N/m

C. 7.3 × 10–2 N

D. 7.3 × 102 N

11

GO ON TO THE NEXT PAGE.

Page 13: Full length 7

Passage VI (Questions 33–38)

Corrosivity is a chemical property that reflects a liq-uid’s ability to dissolve other materials. The Environmen-tal Protection Agency (EPA) defines a corrosive liquid asone that will dissolve steel at a rate of at least 6.35 mm peryear at 55 and has a pH either below 2.0 or above 12.5. Theconcept of pH is fundamental to corrosion because it is thehydronium and hydroxide ions in aqueous solutions thatreact with metals to dissolve them.

The corrosive power of a substance is twice dependenton temperature. First, the rate of a corrosion reaction ispositively correlated to increases in temperature. Second,the pH of a strongly alkaline solution changes with tem-perature. Table 1 lists the changes in pH between 0°C and30°C, around the regulatory limit of pH = 12.5, for anaqueous solution with a large OH– concentration. Thetable shows that as temperature decreases, the pHincreases past the 12.5 limit, but the pOH stays the same.Hence, while the power of a solution to dissolve anothersubstance decreases when the temperature is decreasing,the increase in pH eventually beyond the 12.5 mark wouldmake a so-called “noncorrosive” substance become “cor-rosive” at a lower temperature—contrary to the chemistryinvolved.

Table 1 Data for Acid-Base Changes with Temperature

The corrosion process involves a spontaneous redoxreaction at the surface of a substance, such as a metalexposed to a liquid. For corrosion to occur, all that is nec-essary is an electric potential difference between twopoints on the surface of the metal (due to naturally occur-ring imperfections in the crystal lattice) and some chargecarriers in the liquid in contact with the metal. Elementssuch as gold and platinum resist corrosion very well whileothers like magnesium corrode very easily. For a solidmetal immersed in a solution of its own ions, a higher rel-ative concentration of ions will create a higher relativepotential at the point of contact with the metal.

33. The Standard Hydrogen Electrode, with a half-reac-tion of 2H+(aq) + 2e– → H2(g) and standard reduc-tion potential defined as 0.0 V, is the reference fromwhich the cell potential, E, is measured for all otherstandard reduction reactions. Relative to this hydro-gen electrode, Au and Mg should have standard halfreactions with:

A. a relatively large positive value of E for Au anda relatively large negative value of E for Mg.

B. a relatively large negative value of E for Au anda relatively large positive value of E for Mg.

C. a relatively large positive value of E for Au anda relatively small positive value of E for Mg.

D. relative E values that cannot be predicted fromthe given information and must be determinedby experiment.

34. Which of the following best explains why the pHchanges significantly from 30°C to 0°C in Table 1while the pOH does not?

A. The small concentration of H+ ions from waterself-ionization changes significantly with tem-perature, while the large OH– concentration isunaffected by a small change in the amount ofOH– from water.

B. The excess concentration of OH– reacts morereadily with available H+ at higher tempera-tures, reducing the H+ level.

C. The pH of any aqueous solution will changewith temperature at any concentration of OH–;this effect is not confined to the conditionsshown in Table 1.

D. The pH measures only the amount of H+ and isunaffected by the presence or absence of OH– insolution.

35. A metal surface in contact with water undergoes cor-rosion at pH = 2.0. One possible set of reactions atthe cathode and anode, respectively, would be:

A. Fe(s) → Fe2+(aq) + 2e–;O2(g) + H2O(l) → 4OH–(aq) + 4e–

B. Fe2+(aq) + 2e– → Fe(s); 1/2H2(g) → H+(aq) + e–

C. H+(aq) + e– → 1/2H2(g); Fe(s) → Fe2+(aq) + 2e–

D. Fe2+(aq) + 2e– → Fe(s); Fe(s) → Fe2+(aq) + 2e–

Temp. ( ) pH pOH ](M) pKw

0 13.44 1.50 0.032 14.95 13.23 1.50 0.032 14.715 12.85 1.50 0.032 14.320 12.67 1.50 0.032 14.125 12.50 1.50 0.032 14.030 12.33 1.50 0.032 13.8

°C [OH–

12

GO ON TO THE NEXT PAGE.

Page 14: Full length 7

36. Which of the following statements is NOT consistentwith the passage?

A. The corrosivity of a given solution depends onthe amount of dissolved acid or base in thatsolution.

B. In strongly basic solutions the pH decreaseswith increasing temperature while the pOH doesnot.

C. The corrosivity of a solution increases withincreasing temperature and depends on thekinetic energies of dissolved acid or base in thesolution.

D. The corrosivity of a solution increases withincreasing temperature, while in strongly basicsolutions the corrosivity decreases as the pHdecreases.

37. If the solution in Table 1 was instead set at the regu-latory limit of pH = 2.0 for corrosive acid solutions,which of the following statements would describe thesolution as temperature increases?

A. pOH increases, and pKw increases.

B. pOH decreases, and pKw decreases.

C. pOH decreases, and pKw increases.

D. pOH increases, and pKw decreases.

38. A piece of copper wire which is undergoing corro-sion in a strongly basic solution can be classified asa(n):

A. electrolytic cell with the cathode at negative rel-ative potential, and electrons flowing from cath-ode to anode through the metal.

B. galvanic cell with the cathode at positive rela-tive potential, and electrons flowing from cath-ode to anode through the metal.

C. galvanic cell with the cathode at positive rela-tive potential, and Cu ions being reduced at thecathode.

D. electrolytic cell with the cathode at positive rel-ative potential, and electrons flowing away fromthe anode through the metal.

13

GO ON TO THE NEXT PAGE.

Page 15: Full length 7

Passage VII (Questions 39–43)

Astrophysicists believe that stars produce energy bynucleosynthesis:

4p → � + 2e+ + 2υ + 2γ

Equation 1

where υ represents a very small, neutral and massless par-ticle called a neutrino. This release of energy creates anoutward pressure within a star that counteracts the effectsof gravity. Stars with masses that are less than 1.4 times themass of the Sun will eventually lose their energy and burnout. Starts with masses more than 1.4 times the mass of theSun will eventually contract, with the heavier stars eventu-ally forming a neutron star. Neutron stars, which areessentially a high-density collection of neutrons, have radiiof 10 km, and rotate very rapidly.

As the neutron star rotates, it emits jets of radiation. As theradiation beams periodically sweep past the Earth, theycan be detected with radio telescopes. Neutron stars whoseperiodic emissions are detected on Earth are called pulsars.

(Note: The gravitational constant G = 6.7 × 10–11

N·m2/kg2, the speed of light c = 3 × 108 m/s, and Planck’sconstant h = 4.2 × 10–15 eV·s.)

39. If a 10-kg block is placed on the surface of a neutronstar with a mass of 3.0 × 1030 kg, what is the approx-imate gravitational acceleration of the block?

A. 2.0 × 109 m/s2

B. 2.0 × 1012 m/s2

C. 6.0 × 1012 m/s2

D. 2.0 × 1013 m/s2

40. For a neutron star rotating at a frequency of 450 Hzand emitting jets of radiation, approximately howmuch time passes between consecutive jet emissionsreaching the Earth?

A. 2.2 ms

B. 4.5 ms

C. 22 ms

D. 45 ms

41. In order to conserve charge in Equation 1, the alpha(α) particle must contain:

A. two gamma particles.

B. two neutrinos.

C. two protons.

D. two electrons.

42. If a photon (gamma particle) with energy of 1 × 1016 eVis emitted from a neutron star, what is its wavelength?

A. 1.25 × 10–22 nm

B. 1.25 × 10–13 nm

C. 4.15 × 10–31 nm

D. 8.00 × 1021 nm

43. For an asteroid in a circular orbit about the Sun,which of the following is true?

A. The asteroid’s acceleration is proportional to itsspeed and distance from the Sun.

B. The asteroid’s acceleration is proportional to itsspeed and inversely proportional to its distancefrom the Sun.

C. The asteroid’s acceleration is proportional to thesquare of its speed and inversely proportional toits distance from the Sun.

D. The asteroid’s acceleration is proportional to thesquare of its speed and its distance from the Sun.

14

GO ON TO THE NEXT PAGE.

Page 16: Full length 7

Passage VIII (Questions 44–49)

In 1995, the Galileo orbiter passed by one of Jupiter’smoons, Io, and detected ion cyclotron waves that gavedirect supporting evidence for the formation of SO+

2 ions inIo’s exosphere. Then in 1999, the Galileo orbiter passed byIo in a path angled closer to the surface of the moon. Thistime the presence of SO+ and a small but significantamount of S+ was detected from the ion cyclotron waves.The SO+ emissions were observed closer to the surface ofthe moon, while the SO+

2 emissions were further out in theexosphere. Scientists have concluded from this informa-tion that the exosphere of Io is spatially inhomogeneous.

The magnetized plasma flowing around Io producesions that have velocities predominantly perpendicular tothe moon’s magnetic field and that have a free energy suf-ficient to generate cyclotron waves. The waves aredetected by the spacecraft instrumentation at frequenciesranging from 0.01 to 1.5 Hz, far below the visible spec-trum. The gyrofrequencies of the observed sulfur-contain-ing ions, S+, SO+, and SO+

2, were observed at 0.92 Hz, 0.61Hz, and 0.46 Hz, respectively.

Researchers have discovered that before a volcanoerupts, emission of SO2 gas increases directly above thevolcano’s surface. This observation has led scientists toattribute the detection of S+ and SO+ on the surface of Ioto active volcanic regions. (Note: Planck’s constant, h =6.626 × 10–34 J·s, Rydberg’s constant, R = 8.206 × 10–2

L·atm/(K·mol), and the speed of light, c = 2.998 × 108 m/s)

44. Which of the following sulfur species has the highestoxidation number for the sulfur atom?

A. S+

B. SO+

C. SO+2

D. SO2

45. In what region of the electromagnetic spectrum arethe gyrofrequencies of S+, SO+, and SO+

2 located?

A. X-ray

B. Radio

C. Ultra-violet

D. Gamma ray

46. The energies of the SO+ and SO+2 cyclotron waves,

respectively, are:

A. 4.0 × 10–34 J and 4.0 × 10–34 J.

B. 4.0 × 10–34 J and 6.1 × 10–34 J.

C. 3.3 × 10–25 J and 4.3 × 10–25 J.

D. 6.1 × 10–34 J and 4.0 × 10–34 J.

47. How would the pressure of Io’s exosphere differ fromthat of an ideal gas?

A. The exosphere pressure would be lower, due tothe Coulombic forces between the cations.

B. The exosphere pressure would be lower, due tothe different identities of the gas particles.

C. The exosphere pressure would be higher, due tothe different identities of the gas particles.

D. The exosphere pressure would be higher, due tothe Coulombic forces between the cations.

48. Considering kinetic molecular theory, which one ofthe following would NOT be correct regarding theexosphere of Io?

A. The exosphere of Io consists of gas particles inwhich the distances between the particles arelarge compared to the radii of the particles.

B. The ions in the exosphere of Io exhibit an inher-ent pressure as the result of random collisionsbetween the ions in motion.

C. The ions of Io’s exosphere have an averagekinetic energy proportional to the temperature.

D. The collisions between the ion in the exosphereof Io are elastic collisions with no repulsiveforces between the ions during collision.

49. A mixture of positively charged ions and electrons iscalled:

A. a cyclotron wave mixture.

B. a neutral soup.

C. a plasma.

D. a quadrupole.

15

GO ON TO THE NEXT PAGE.

Page 17: Full length 7

50. A crane lifts a 1000-kg steel beam off the ground,and sets it down on scaffolding 100 meters off theground. All of the following are true EXCEPT:

A. The net work done on the steel beam is 9.8 × 105 J.

B. The net work done on the steel beam is 0 J.

C. The magnitude of the work done on the steelbeam by gravity is 9.8 × 105 J.

D. The magnitude of the work done on the steelbeam by the crane is 9.8 × 105 J.

51. At 60°C in pure water, what is true about the valuesof Kw, [H+], and [OH–]?

A. Kw > 10–14, [H+] = [OH–] > 10–7

B. Kw > 10–14, [H+] > 10–7, [OH–] > 10–7

C. Kw < 10–14, [H+] = [OH–] > 10–7

D. Kw < 10–14, [H+] > 10–7, [OH–] > 10–7

52. If an object is located farther from a concave mirrorthan the mirror’s focal point, what must be true aboutthe image?

A. The image is real and upright.

B. The image is real and inverted.

C. The image is virtual and upright.

D. The image is virtual and inverted.

53. For a reaction,

AB2(l) + 2C(g) + heat → 2BC(g) + A(g)

at a temperature of 800 K, the forward reactionwould be best characterized as:

A. spontaneous.

B. non-spontaneous.

C. at equilibrium.

D. cannot be determined from information given.

54. A 3.00-kg block is given an initial speed of 4.00 m/sdown an inclined plane, as shown. The force on theblock due to friction from the plane is a constant 5.00N. What is the kinetic energy of the block when itreaches the bottom of the plane?

A. 5.93 J

B. 30.0 J

C. 52.8 J

D. 82.8 J

55. Which quantum number affects an atomic orbital’soverall shape?

A. n

B. l

C. s

D. ms

d = 6 m

h =

2 m

16

GO ON TO THE NEXT PAGE.

Questions 50 through 55 are NOT based on adescriptive passage.

Page 18: Full length 7

Passage IX (Questions 56–61)

Figure 1 shows an electromagnetic apparatus that isdesigned to determine the sign of the charge carriers flow-ing through a conducting strip. Consider a conductingmetal strip of length L, width w and depth d, with conven-tional current I running along the length of the strip. Thestrip is placed in a uniform magnetic field B, which is per-pendicular to the strip and permeates the metal. The mag-netic field exerts a force on the charge carriers that flowthrough the strip.

As the charge carriers build up on one side of the strip,a charge separation is created, causing an electric field tobe formed between the strip’s two edges. This process iscalled the Hall effect. Over time, the system reaches anequilibrium in which the force on the charge carriers dueto the magnetic field is balanced by the force due to theelectric field. By using a voltmeter to measure the poten-tial difference between points a and b (the Hall voltage),the direction the magnetic force on the charge carriers andthe sign of the charge carriers can be determined.

Figure 1

It is possible to use this apparatus to perform experi-ments to determine the charge density (n) within differentconducting strips. Charge density is defined as the numberof charge carriers per unit volume of the strip. Table 1 pro-vides data collected from several experiments that use theHall voltage apparatus to determine charge density. Themagnitude of the charge carriers is the same in each strip.

Table 1 Data for the Charge Density of a Conducting Strip

56. Assume the charge carriers are electrons. What istrue about the Hall voltage between points a and b?

A. The potential at point a is higher than the poten-tial at point b, because the electrons are drawn topoint a.

B. The potential at point a is lower than the poten-tial at point b, because the electrons are drawn topoint a.

C. The potential at point a is higher than the poten-tial at point b, because the electrons are drawn topoint b.

D. The potential at point a is lower than the poten-tial at point b, because the electrons are drawn topoint b.

57. At equilibrium, assume the magnitude of the mag-netic field is 2.6 T, the width of the strip is 1.7 cm andthe voltage measured across the strip is 11.1 µV.What is the average speed of the charge carriers inthe strip?

A. 2.51 × 10–4 m/s

B. 6.53 × 10–6 m/s

C. 1.96 × 10–6 m/s

D. The charge carriers are not moving at equilib-rium.

58. After equilibrium is reached, the magnetic field isturned off. What happens to the charge carriers?

A. Their velocity is zero.

B. Their velocity remains constant.

C. They accelerate.

D. They gain potential energy.

w

VH

B

a b

I

17

GO ON TO THE NEXT PAGE.

Strip 1 Strip 2 Strip 3

B (T) 1.2 1.2 1.2

VH (µV) 6.2 6.2 6.2

I (A) 1.8 1.8 3.7

w (mm) 10 20 20

d (mm) 1.5 1.5 1.5

n (m–3) 1.45 × 10–27 1.45 × 10–27 2.89 × 10–27

Page 19: Full length 7

59. According to the data presented in Table 1, which ofthe following could represent the relationshipbetween current density n, current I and width w?

A. n is proportional to w/I.

B. n is inversely proportional to I and independentof w.

C. n is inversely proportional to w and independentof I.

D. n is proportional to I and independent of w.

60. If strip 1 (from Table 1) were 3-cm in length, whatwould be the magnitude of the net force on it due tothe magnetic field?

A. 6.48 × 10–2 N

B. 3.35 × 10–1 N

C. 2.16 N

D. 6.48 N

61. Imagine that instead of pointing into the page, themagnetic field B points directly to the left, along thewidth of the strip. Which statement best characterizesthis new arrangement?

A. The behavior of the charge carriers does notchange from that in the original apparatus.

B. An electric field is not generated across thewidth of the strip.

C. The electric field points in the opposite direc-tion.

D. A Hall voltage is not created anywhere in thestrip.

Passage X (Questions 62–66)

In order to explain the physical properties of metallicmaterials, band theory is invoked. When single atomscombine to form molecules, the valence orbitals of theatoms combine with each other to form molecular orbitals.The arithmetic sum of two orbitals is a “bonding” orbital,with lower energy, and the difference between two orbitalsis an “antibonding” orbital, with higher energy. As moreand more atoms join together to form a metallic crystal, thenumber of orbitals grows linearly. A macroscopic crystalthus has a huge number of molecular orbitals—each witha microscopically discrete energy level and unique degreeof bonding and antibonding character—available for elec-trons to fill. This essentially continuous series of orbitals iscalled a “band.” Thus N valence orbitals combine to createa band with 2N vacancies for electrons. Figure 1 shows aschematic diagram for constructing a band for a Group IAmetal.

Figure 1

A completely filled band is known as a valence band;an empty or partially-filled band is a conduction band. Theconduction band allows electrons to jump easily from atomto atom in response to a potential difference: hence thehigh electrical conductivity of metals. Additionally, if themetal is deformed under external stress, the band can eas-ily readjust its component orbitals and their energiesslightly, shifting the electrons around with little difficulty;this is why metals are malleable.

Bands in metals can be constructed from s orbitals, porbitals, and d orbitals, creating s bands, p bands, and dbands, respectively. In metals, the different band types mayhave continuous or even overlapping energy ranges. Innonmetals, there is an energy separation between the dif-ferent bands known as a band gap.

1 2 3 4 N

Electrons Fill Portion of Band

Number of Atoms, N, Combined

Ene

rgy

18

GO ON TO THE NEXT PAGE.

Page 20: Full length 7

62. Given 1.5 mol Na, how many valence orbitals willcombine to form a band?

A. 6.0 × 1023

B. 9.0 × 1023

C. 1.8 × 1024

D. 1.5 × 1023

63. Based on information in the passage, which is themost likely band structure for calcium?

64. Which of the following is a property of solid sulfur?

A. Thermally insulating

B. Malleable

C. Electrically conductive

D. Alloy forming

65. For a 3d electron, which value of m1 is impossible?

A. 0

B. 1

C. 2

D. 3

66. At temperatures above absolute zero, the population,P, of a discrete orbital is given by the Fermi-Diracdistribution:

P =

where E is the energy of the orbital, EF is the FermiEnergy, and k is the Boltzmann constant. What is thepopulation of an orbital at the Fermi Energy at 298 K(k = 1.38 × 10–23 J·K–1)?

A. 0

B. 1

C. 0.5

D. Cannot be determined from information given.

1��e(E–EF)/kT + 1

A.

4s band

Ene

rgy

3d band

B.

4s band

Ene

rgy

3p band

C.

3p band

Ene

rgy

3s band

D.

3d band

Ene

rgy

4s band

19

GO ON TO THE NEXT PAGE.

Page 21: Full length 7

Passage XI (Questions 67–72)

A chemistry student analyzed vapor pressure changesin solution. In a closed container at 50°C, the student pre-pared solutions composed of two nonvolatile liquids,Compound A and Compound B. For various solution com-positions, the student measured the total vapor pressure(Pt) and the vapor pressure of each solution component(PA, PB). She recorded these results as solid lines in Figure1 below. After completing the experiment, the student cal-culated the vapor pressure for each component as if itbehaved as part of an ideal solution at various mole frac-tions. She recorded these values in Figure 1 as dashedlines. The vapor pressures of pure Compounds A and B are386 and 812 mm Hg, respectively.

Figure 1

67. The student’s measurements would more closelyadhere to the ideal solution prediction if:

A. Compounds A and B were more polar.

B. the solution was more dilute.

C. she had conducted her experiment at higherpressures.

D. Compounds A and B had more similar chemicalproperties.

68. The student found that the vapor pressure of Com-pound B was significantly lowered after addition of anonvolatile solute—Compound C. This phenomenonis a result of:

A. disproportionation.

B. Hund’s rule.

C. colligative properties.

D. the ideal gas law.

69. At a mole fraction of 0.600 for Compound A, themeasured vapor pressure of Compound B is:

A. 525 mm Hg.

B. 273 mm Hg.

C. 213 mm Hg.

D. 105 mm Hg.

70. An ideal solution is 0.70 mole fraction Compound Band 0.30 mole fraction Compound A. The total vaporpressure above the solution is:

A. 684 mm Hg.

B. 834 mm Hg.

C. 576 mm Hg.

D. 552 mm Hg.

71. A solution is 0.80 mole fraction Compound B and0.20 mole fraction Compound A. Which statement iscorrect regarding the vapor above the solution?

A. Compound A has a higher mole fraction in thevapor phase than in solution.

B. Compound B has a higher mole fraction in thevapor phase than in solution.

C. Compound A has a higher partial pressure thanthe total vapor pressure.

D. Compound B has a higher partial pressure thanthe total vapor pressure.

72. At 0.4 mole fraction Compound A in ideal solution,the vapor pressure of Compound A will have beenlowered by:

A. 232 mm Hg.

B. 108 mm Hg.

C. 487 mm Hg.

D. 150 mm Hg.

1000

800

600

400

200

00 0.2 0.4 0.6 0.8 1.0

P° = pureCompound A

Pt

PB

PA

P° = pureCompound B

Vap

or P

ress

ure

(mm

Hg)

Mole Fraction Compound B

B

A

20

GO ON TO THE NEXT PAGE.

Page 22: Full length 7

73. A –7 µC charge experiences a repulsive 500-N forcewhen placed 2 cm away from a second charged par-ticle. What is the charge of the second particle?

(k = 9 × 109 N·m2/C2)

A. –3.2 µC

B. +3.2 µC

C. –0.16 nC

D. +0.16 nC

74. The standard reduction potential of Au3+(aq)+3e– → Au(s) is 1.498 V.

The standard reduction potential of 2S(s) + 2e– → S2–

2 (aq) is –0.428V.

Based on this information, it must be true that:

A. S2–2 is more easily oxidized than Au, and Au is

thus a better oxidizing agent than S2–2 .

B. S2–2 is more easily oxidized than Au, and Au is

thus a better reducing agent than S2–2 .

C. S2–2 is more easily reduced than Au, and Au is

thus a better oxidizing agent than S2–2 .

D. S2–2 is more easily reduced than Au, and Au is

thus a better reducing agent than S2–2 .

75. An 8-kg object traveling at 5 m/s has a perfectlyinelastic collision with a stationary 13-kg object.What is the ratio of the initial speed of the 8-kgobject to its final speed?

A. 8:21

B. 21:40

C. 13:8

D. 21:8

76. Which of the following atoms has the smallestdiameter?

A. Al

B. B

C. C

D. N

77. If a thermodynamic system undergoes a process inwhich it does 1300 J of work on its surroundings, butits internal energy increases by 700 J, which of thefollowing must be true?

A. The system loses 600 J of heat energy.

B. The system gains 600 J of heat energy.

C. The system loses 2000 J of heat energy.

D. The system gains 2000 J of heat energy.

STOP. IF YOU FINISH BEFORE TIME IS CALLED,CHECK YOUR WORK.YOU MAY GO BACK TO ANYQUESTION IN THIS SECTION ONLY.

21

Questions 73 through 77 are NOT based on adescriptive passage.

Page 23: Full length 7
Page 24: Full length 7

Verbal ReasoningTime: 85 MinutesQuestions 78–137

DO NOT BEGIN THIS SECTION UNTIL YOU ARE TOLD TO DO SO.

Page 25: Full length 7

Passage I (Questions 78–83)

There can be nothing simpler than an elementary par-ticle: it is an indivisible shard of matter, without internalstructure and without detectable shape or size. One mightexpect commensurate simplicity in the theories thatdescribe such particles and the forces through which theyinteract; at the least, one might expect the structure of theworld to be explained with a minimum number of particlesand forces. Judged by this criterion of parsimony, adescription of nature that has evolved in the past severalyears can be accounted a reasonable success. Matter isbuilt out of just two classes of elementary particles: theleptons, such as the electron, and the quarks, which areconstituents of the proton, the neutron and many relatedparticles. Four basic forces act between the elementaryparticles. Gravitation and electromagnetism have longbeen familiar in the macroscopic world; the weak forceand the strong force are observed only in subnuclearevents. In principle this complement of particles and forcescould account for the entire observed hierarchy of materialstructure, from the nuclei of atoms to stars and galaxies.

An understanding of nature at this level of detail is aremarkable achievement; nevertheless, it is possible toimagine what a still simpler theory might be like. The exis-tence of two disparate classes of elementary particles is notfully satisfying; ideally, one class would suffice. Similarly,the existence of four forces seems a needless complication;one force might explain all the interactions of elementaryparticles. An ambitious new theory now promises at least apartial unification along these lines. The theory does notembrace gravitation, which is by far the feeblest of theforces and may be fundamentally different from the others.If gravitation is excluded, however, the theory unifies allelementary particles and forces.

The first step in the construction of the unified theorywas the demonstration that the weak, the strong, and theelectromagnetic forces could all be described by theoriesof the same general kind. The three forces remained dis-

tinct, but they could be seen to operate through the samemechanism. In the course of this development a deep con-nection was discovered between the weak force and elec-tromagnetism, a connection that hinted at a still grandersynthesis. The new theory is the leading candidate foraccomplishing the synthesis. It incorporates the leptonsand the quarks into a single family and provides a meansof transforming one kind of particle into the other. At thesame time the weak, the strong, and the electro-magneticforces are understood as aspects of a single underlyingforce. With only one class of particles and one force (plusgravitation), the unified theory is a model of frugality.

78. All of the following are differences between the twotheories described by the author EXCEPT:

A. the second theory is simpler than the first.

B. the first theory encompasses gravitation whilethe second does not.

C. the second theory includes only one class of ele-mentary particles.

D. the first theory accounts for only part of thehierarchy of material structure.

79. According to the passage, which of the following aretrue of quarks?

I. They are the elementary building blocksfor neutrons.

II. Scientists have described them as havingno internal structure.

III. Some scientists group them with leptons ina single class of particles.

A. I only

B. III only

C. I and II only

D. I, II, and III

24

GO ON TO THE NEXT PAGE.

VERBAL REASONING

DIRECTIONS: There are nine passages in the VerbalReasoning test. Each passage is followed by severalquestions. After reading a passage, select the bestanswer to each question. If you are not certain of ananswer, eliminate the alternatives that you know to beincorrect and then select an answer from the remain-ing alternatives. Indicate your selection by blackeningthe corresponding oval on your answer document.

5

10

15

20

25

30

35

40

45

Page 26: Full length 7

80. The author considers which of the following in judg-ing the usefulness of a theory of elementary particlesand forces?

I. the simplicity of the theory

II. the ability of the theory to account for thelargest possible number of known phenom-ena

III. the possibility of proving or disproving thetheory by experiment

A. I only

B. II only

C. I and II only

D. I and III only

81. It can be inferred that the author considers the failureto unify gravitation with other forces in the theory hedescribes to be:

A. a disqualifying defect.

B. an unjustified deviation.

C. a needless oversimplification.

D. an unavoidable limitation.

82. Which of the following best describes the organiza-tion of the passage?

A. It enumerates distinctions among several differ-ent kinds of elementary particles.

B. It states a criterion for judging theories ofnature, and using it to evaluate two theories.

C. It explains three methods of grouping particlesand forces.

D. It criticizes an inaccurate view of elementalnature and proposing an alternative approach.

83. It can be inferred that the author would be likely toconsider a new theory of nature superior to presenttheories if it were to:

A. account for a larger number of macroscopicstructures than present theories.

B. reduce the four basic forces to two more funda-mental, incompatible forces.

C. propose a smaller number of fundamental parti-cles and forces than current theories.

D. successfully account for the observable behav-ior of bodies due to gravity.

25

GO ON TO THE NEXT PAGE.

Page 27: Full length 7

Passage II (Questions 84–90)

In the United States, the majority of young childrenwhose first language is English have no familiarity with asecond language, unlike continental-European childrenwho are required to study a second language from a veryyoung age. Unfortunately, this type of early childhoodSecond Language Acquisition (SLA) is largely neglectedin the US, even though studies have shown that childrenacquire communicative language skills faster and moreaccurately during their formative kindergarten and ele-mentary-school years. Children acquire their own nativelanguage in this way, and can just as easily acquire a sec-ond language during this same time. Schools in the USshould be required to institute early SLA programs begin-ning at the kindergarten level to assure that childrenbecome familiar with at least one foreign language.

Familiarity with a foreign language at a young ageexposes children to other cultures, which in turn fosters achild’s acceptance of differences. Claims have been madethat cultural knowledge can be just as successfullyacquired in English, and that SLA programs are thereforenot necessary. But how can children fully appreciateanother culture if they have no familiarity with that cul-ture’s communication system? Since languages developedas societies developed, the very foundations of a cultureare deeply embedded in its lexicon. Even native speakersof a language may be oblivious to moral, historical, politi-cal, and cultural subtleties in the words they speak, all ofwhich may cause misunderstandings based on the unspo-ken influences lurking behind the spoken word. However,the earlier a child learns a second language, the more thatchild will learn to appreciate, rather than disdain, culturaldifferences. As this child grows older, this appreciationwill help to prevent cultural or linguistic misunderstand-ings with native speakers of the child’s non-native lan-guage.

Introducing a foreign language at a young age teacheschildren that there is a vast world of cultural experiencebeyond their own, and they learn to appreciate this diver-sity rather than to fear it irrationally. To increase the possi-bility of leaving our children the legacy of a peacefulworld, we need to teach them to appreciate and embraceother cultures, a lesson that can be accomplished mosteffectively through early childhood SLA programs.

Opponents of early SLA programs cite additionalexpenses (for new curriculum and new SLA teachers) asobstacles to early SLA programs. Opponents also arguethat SLA is not one of the “basic skills” like math, reading,and writing, which should be the focus of children’s earlylearning, and that SLA could, in fact, detract from a child’s

acquisition of English-language skills. Successful SLA,however, requires a solid knowledge of one’s own nativelanguage, so that linguistic characteristics, like grammarand structure, can be fully understood. In this way, speak-ing a second language would actually augment a child’scommunicative skills in English. In addition, SLA pro-grams can be integrated into the classroom as part of read-ing and writing units modified to include lessons in bothEnglish and another language. Since those training to beteachers are required to have some proficiency in a secondlanguage in order to earn their certification or their Mastersdegree, they could capitalize on this knowledge by expos-ing their students to a foreign language and culture in theirclassrooms. Thus the fear of additional expenses andneglect of English skills are not valid arguments againstearly SLA programs. Requiring SLA programs in kinder-garten would clearly benefit native English-speaking chil-dren in the US, and could, in the long run, even have apositive effect on international relations in general.

84. With which of the following statements would theauthor most likely agree?

A. Being fluent in more than one language isuncommon only in the United States.

B. Since children in most countries have to learnEnglish, there is no reason for English-speakingchildren to learn a foreign language.

C. Increasing the number of bilingual children inthe United States would have a beneficialimpact on foreign relations in the future.

D. All bilingual people have a firm grasp of the cul-tural and societal values that underlie their sec-ond language.

85. The author employs which of the following to refutethe contention that “cultural knowledge can be…suc-cessfully acquired in English?”

A. support from an authority who denies the statedclaim and agrees with the author

B. data from a study that proves this contention isinvalid

C. subjective criticism rather than objective analy-sis of the contention

D. formation of a rhetorical question that castsdoubt on the argument

26

GO ON TO THE NEXT PAGE.

5

10

15

20

25

30

35

40

45

50

55

60

65

Page 28: Full length 7

86. The author’s conclusion that non-bilingual childrenare at a disadvantage when compared to children whoare required to learn a second language depends onall of the following assumptions EXCEPT:

A. children who learn a foreign language at anearly age become fluent in their second lan-guage.

B. non-bilingual children are considered less intel-ligent than their bilingual counterparts.

C. youths who begin study of a foreign language intheir pre-teen years have more difficulty becom-ing fluently bilingual.

D. children will appreciate different cultures ifSLA begins at the pre-school stage.

87. Based on the author’s statements about the relation-ship between a culture and its language, which of thefollowing activities would the author most likelyprefer?

A. translation of a lullaby from the foreign lan-guage into English with illustrations of the lul-laby’s words

B. memorization of situation-based foreign wordsplaced in a context appropriate to a similar situ-ation in the US

C. performance of a ritual celebration with appro-priate costumes, props, and vocabulary in thesecond language, with minimal explanations inEnglish

D. a project to find holidays celebrated in the for-eign country that are equivalent to holidays cel-ebrated in the US

88. According to the passage, which of the following hasbeen a factor of international conflicts?

A. lack of accurate communication between speak-ers of different languages

B. illogical fear of different cultures or peoplesbased on ignorance

C. intentional offenses committed by one cultureagainst another

D. disagreement over international policies andlaws

89. To refute concerns raised by opponents to early SLAprograms, the author:

A. presents testimonials from kindergarten teach-ers to refute the objections.

B. makes a blanket statement that the objectionsare false but fails to provide support for thisclaim.

C. analyzes each objection individually and thenprovides examples to contradict each objection.

D. describes these concerns and presents a counter-argument to each.

90. What is the author’s purpose in referring to “the lex-icon of each language” in paragraph 2?

A. to lead into a discussion of how cultural refer-ents are an integral and invisible part of lan-guage

B. to support the belief that translations intoEnglish may not be completely accurate

C. to show how subtle cultural meanings of a lan-guage may be misinterpreted by non-nativespeakers

D. to demonstrate that different cultures cannot beappreciated without familiarity of the culture’slanguage

27

GO ON TO THE NEXT PAGE.

Page 29: Full length 7

Passage III (Questions 91–97)

Given his luminous treatment of light, sky, and water,J.M.W. Turner (1775–1851) is often viewed in hindsight asa precursor of Impressionism. Yet as Turner authorityAndrew Wilton has argued, his roots lie in a specificallyeighteenth-century tradition, that of the “sublime.” Beforelandscape painting was accepted in England as the rendi-tion of everyday reality, it was seen as the expression of astate of spiritual exaltation.

The roots of the notion of the sublime, Wilton notes, goback to antiquity: Longinus observed (according to aneighteenth-century paraphrase) that “the effect of the sub-lime is to lift up the soul...so that participating, as it were,of the splendors of the divinity, it becomes filled with joyand exultation.” The sublime, therefore, was understood toproduce an effect of elevation toward unity with the divine.

In its origins, the sublime was associated with literaryrather than visual art, as its connotations of power andmystery could most easily be conveyed in words; and itssubject matter was epic, historical, or religious. To eigh-teenth-century commentators, Homer, the Bible, and Mil-ton were quintessentially sublime. When the concept wasapplied to painting, this narrative emphasis was main-tained, leading almost by necessity to a focus on thehuman figure; for Joshua Reynolds, Michelangelo’s Sis-tine Chapel frescoes exemplified the sublime in art.Because it did not show figures (except incidentally) land-scape was necessarily seen as inferior.

The transition to the conception that produced Turner’slandscapes had several sources. One was the eighteenthcentury’s quasi-religious excitement in scientific investiga-tion of nature, shown for example when Addisonexclaimed upon the astronomer’s “pleasing astonishment,to see so many worlds, hanging one above another, andsliding round their axles in such an amazing pomp andsolemnity.” A second was the rise of a middle class withthe leisure to travel, which led to an interest in the ruggedvistas of Wales and Scotland. Finally, James Thomson’simmensely popular nature epic “The Seasons” (1726–30)applied blank verse, with its connotations of loftiness, tothe portrayal of nature’s immensities.

By the latter part of the century, there was a well-defined notion of the sublime in literature and painting,which included nature while by no means excluding earlierreferents. According to Edmund Burke’s definitive essayof 1757, the sublime in nature was closely tied up withvastness, lack of habitation and cultivation, and dangerwhich, as in the reaction to high mountain passes or stormsat sea, was conducive to awe. These qualities, as evoked in

the painting of landscapes (and urban vistas, an importantthough subordinate field), produced a series of genres that,Wilton stresses, form the key to Turner’s work: the “pic-turesque sublime,” the “terrific” (wild crags, cataracts,etc.), the sublime of the sea, mountains, and darkness, andfinally the “architectural sublime” and the urban sublime.

91. Based on the information in the passage, which of thefollowing is LEAST likely to have been the subjectof a painting of the sublime?

A. a cathedral in the center of a city

B. a storm at sea

C. the eruption of a volcano

D. wheat fields by a country road

92. According to the author, Burke contributed to thedevelopment of the concept of the sublime by:

A. classifying the genres of the sublime in art.

B. broadening the conception of the sublime toinclude nature.

C. giving a more clear-cut definition of the sublimethan earlier writers.

D. defining some of the qualities in nature thatcould be considered sublime.

28

GO ON TO THE NEXT PAGE.

5

10

15

20

25

30

35

40

45

50

Page 30: Full length 7

93. According to the passage, which of the followinghelped change the conception of the sublime in theeighteenth century?

I. Depiction of nature through a literary formassociated with noble sentiments

II. Addison’s popularization of recent discov-eries in astronomy

III. Socio-economic changes in English society

A. I only

B. III only

C. I and II

D. I and III

94. The author suggests that as the concept of the sub-lime developed:

A. it lost its connotations of closeness to divinity.

B. historical and epic subjects were no longer seenas embodying the sublime.

C. connotations previously restricted to the humanfigure were applied to natural scenes.

D. painters ceased to see Biblical stories as appro-priate subjects.

95. In the third paragraph, the author suggests that the“rise of a middle class with the leisure to travel”:

A. explains the popularity of Turner’s Welsh andScottish landscapes.

B. led to increased popular interest in the sublime.

C. clarifies the origins of Turner’s interest in natu-ral subjects.

D. influenced cultural sensibilities toward nature.

96. According to the passage, landscapes were not origi-nally seen as embodying the sublime because:

A. the narrative connotations of the sublimeimplied an emphasis on the human figure.

B. only religious subjects were seen as embodyingthe sublime.

C. Michelangelo did not paint landscapes.

D. landscape was viewed purely as the visual rep-resentation of everyday nature scenes.

97. The author gives specific examples of all of the fol-lowing EXCEPT:

A. the conception of the sublime held in antiquity.

B. the subject matter which might be considered asrepresenting the “terrific.”

C. a work of visual art considered as embodyingthe sublime by an eighteenth century authority.

D. a conception similar to that of the sublime in anon-artistic context.

29

GO ON TO THE NEXT PAGE.

Page 31: Full length 7

Passage IV (Questions 98–103)

Although there has been some debate about the properetymology of the word retina, this author believes that thecorrect etymology is from the Latin “rete.” It is not clear,however, whether the word is ultimately a translation ofthe Greek word amphiblestron or the Arabic al-shabakiyyah.

Part of the confusion stems from what part of the eyeHerophilus, a famous physician who practiced in Alexan-dria (c. 325–280 BC), referred to when he named theretina. Since all of Herophilus’ works have been lost, andwe have to rely on quotations of him by later authors, thereis no way to check his words directly. Rufus, another clas-sical physician (fl. AD 98–117), indicated that, “Theancient name by which it is called is cobweb-like (arach-noeides), on account of its fineness. But Herophilus likensit to a casting net that is drawn up, some also call it ‘net-like’. Others call it ‘vitreous,’ too, on the basis of the liq-uid.” On the other hand, Celsus (25 BC–AD 50) writes that“Herophilus called the retina arachnoidem, cobweb-like.”Margaret May, in her translation of Galen’s (130–200) DeUsu Partium, implies that Herophilus changed the namefrom arachnoid to amphiblestron. In most of theseaccounts there is some confusion as to which layer of theeye is being referred to. Perhaps, also, Herophilus usedboth terms.

Assuming that Herophilus referred to this structure asamphiblestron, how do we get from this to “retina”?Amphiblestron derives from amphibello, which means“something thrown around.” In Greek, the word can referto a garment, a net, an encompassment, or a fetter. Accord-ing to the Graceum Lexicon Manuale by A. B. Hederico(1790), amphiblestron has three Latin translations: rete,tunica, and munitio. I suspect that amphiblestron wastranslated “rete” (which means “net”) instead of the moreobvious and accurate “tunica.” From “rete,” it is a shortetymological step to “retina.”

In his Medical Etymology, Perry Pepper combines allthese into a probable etymology of this structure. He says,“[Rete] is probably the correct derivation, although there isno truly net-like structure in the retina. It probably cameabout as follows: Galen applied to the structure the Greekword amphiblestron, which had two relevant meanings: asurrounding coat [tunica] and a fisherman’s net [rete]. Heused the word in the first sense; but when it was translatedinto Latin, the translator chose the second meaning andused the Latin word rete, net.”

Although Pepper’s conjecture is substantially correct(except it was Herophilus, not Galen, who named thestructure), Stephen Polyak argues that rete is a translationof the Arabic word for the retina: al-shabakiyyah (netlike).He argues that this translation is from Gherard of Cre-mona’s translation of al-shabakiyyah in ibn Sina’s al-Qanun (better known in the west as Avicenna’s Canon,980–1037) from Arabic into Latin. Since Avicenna inte-grated a great deal of Greek science into his works, it ispossible that he independently picked an inapt translationof amphiblestron, using the Arabic word for “net.”

At this point, we are left with several unresolvedissues, but we can also make some statements with a highdegree of assurance. Putting aside for the time being thequestion of whether the original term for the retina was“arachnoidem”, it seems clear that the English word retinaderives from the Latin “rete.” It is less clear, however,whether the Latin “rete” is an inapt translation from theGreek amphiblestron or the appropriate translation of theinappropriate Arabic word al-shabakiyyah.

98. Which of the following does the author of the pas-sage conclude is the etymology of the word “retina”?

A. It derives from the Arabic word for net.

B. It derives from the Greek word for net.

C. It derives from the Latin word for net.

D. It is not clear what the etymology is.

99. Given the disputes about the Greek word for theretina, the Arabic word for the retina is importantbecause:

A. it confirms that Herophilus called the retinaamphiblestron.

B. it confirms that Herophilus called the retinaarachnoeides.

C. if retina is derived from the Arabic word, we canbypass the disputes about the Greek word.

D. it shows that Arabic scientists had some knowl-edge of Greek anatomical works.

30

GO ON TO THE NEXT PAGE.

5

10

15

20

25

30

35

40

45

50

55

60

65

Page 32: Full length 7

100. The author calls “rete” an inapt translation of theGreek word for the retina because:

A. rete is not a possible translation of the Greek.

B. tunica would have been a more accurate transla-tion of the Greek.

C. munitio would have been a more accurate trans-lation of the Greek.

D. rete was already used to refer to another struc-ture in the brain.

101. Avicenna is known to have integrated many Greekterms into his writings. Based on the information inthe passage, it is therefore possible that:

A. Avicenna misunderstood the meaning of theGreek word.

B. Avicenna knew the proper translation, butwanted to confuse later writers.

C. Even if we use the Arabic translation for retina,it would still derive from the Greek.

D. There was some confusion about the propertranslation during Avicenna’s time.

102. According to the passage, it might be relevant thatHerophilus called the retina arachnoeides because:

A. it would support the translation of rete overtunica.

B. it would support the translation of tunica overrete.

C. it would support the translation of munitio overrete.

D. it is irrelevant what Herophilus called the retina.

103. According to the author, which of the following arepossible sources of confusion about the etymology ofthe word retina?

I. There is some confusion about the wordHerophilus used to refer to the retina.

II. There is some confusion about what part ofthe eye that Herophilus referred to.

III. There is some confusion about the part ofthe eye that Avicenna referred to.

A. I only

B. I and II

C. II and III

D. I, II, and III

31

GO ON TO THE NEXT PAGE.

Page 33: Full length 7

Passage V (Questions 104–110)

Granted, affirmative action policies have produced cer-tain encouraging statistics—the number of Black profes-sionals is now 250 percent of the 1965 figure—but otherindicators show overall deterioration in the economic con-dition of the majority of Black people since the mid-fifties.At the time of the Civil Rights Act of 1964, the averageBlack family income was 61 percent of that of its whitecounterpart. By 1978, that figure was 50 percent. Unem-ployment rates for young Black males are increasing fasterthan those of any other group; meanwhile, the proportionof Black workers able to find the unskilled jobs tradition-ally available to them is steadily decreasing. William JuliusWilson suggests that these two facts, taken together, fore-shadow the perpetuation of current ghetto conditions.Males with no work experience are unable to secure evenlow-status labor to support themselves and their families;in the past, this has often resulted in their leaving home, forboth emotional and economic reasons. Women thusbecome the heads of families trapped in long-term poverty.

After the Second World War, expansion of the econ-omy and state intervention to promote racial equalityresulted in the eventual improvement of conditions for theBlack middle class. But affirmative action policies do notaddress the structural characteristics of the American econ-omy which keep large numbers of urban Black personsunemployed, namely, the increased use of industrial tech-nology, the relocation of industry out of the cities, and theshift from manufacturing to service industries.

Black scholars of the past two decades have addressedthe macroeconomic conditions which structure the urbanghetto only indirectly, focusing instead on the social insti-tutions thereby engendered. As early as 1967, KennethClark pointed out that “civil rights victories benefited onlya very small percentage of middle class Negroes, while the[predicament of the masses of Black people] remained thesame or worsened.” Clark’s central thesis, however, wasthat the conditions of ghetto life impair the psychologicalability of Black people to function outside its confines.

This analysis was rejected by the intellectual wing ofthe Black Power movement; scholars such as Robert Sta-ples and Nathan Hare refused to see the social structures ofthe ghetto as pathological. But in their valid attempt toremove racist influence from the social sciences, they per-haps missed the point: Overlooking economic reasons forthe predominance of female-headed families in the ghetto,they insisted instead on the origins of the matriarchal pat-tern in Black history and culture.

Robert K. Merton has noted that the collective “needfor self-affirmation” experienced by a newly—empoweredgroup “is a predictable and intensified counter-response tolong—standing belittlement from without.” Nevertheless,we must concur with Orlando Patterson that “Black ethnicchauvinism” has proved to be “a form of mystifica-tion...making it difficult for Blacks to see how their fate istied to the American economy.”

104. It can be inferred that the author believes the“encouraging statistics” produced by affirmativeaction (lines 1–3):

A. are based on an unrepresentative sample ofworking Black people.

B. give the impression of an improvement in con-ditions for Black people in general.

C. are invalidated by more reliable data.

D. represent the status of only non-urban residents.

105. William Julius Wilson’s argument (lines 13–19)depends on which of the following assumptions?

A. Unemployment rates for Black people will con-tinue to rise.

B. Black workers are no longer willing to performlow-status labor.

C. Poor families headed by women have littlesocial or economic mobility.

D. Most families on welfare are headed by females.

32

GO ON TO THE NEXT PAGE.

5

10

15

20

25

30

35

40

45

50

55

Page 34: Full length 7

106. In lines 36–38, the author quotes from Kenneth Clarkin order to indicate that:

A. Clark’s argument exemplifies the misinterpret-ation which prevailed in the late sixties.

B. Clark had some awareness of the class issueswhich the author believes should be empha-sized.

C. Clark was the most effective opponent of theBlack Power movement.

D. Clark emphasized economic causes over psy-chological effects.

107. It can be inferred that scholars associated with theBlack Power movement rejected Kenneth Clark’sanalysis primarily because Clark:

A. underestimated the pervasiveness of racism.

B. dealt insufficiently with economic factors.

C. saw ghetto social patterns as harmful and self-perpetuating.

D. applied essentially racist criteria.

108. The author would most probably agree that theprevalence of female-headed families in the Blackpopulation:

I. is a positive cultural tradition.

II. is part of a larger negative dynamic.

III. has economic origins.

A. II only

B. I and II only

C. I and III only

D. II and III only

109. Judging from the passage, which of the followingbest describes the relationship between the ideas ofRobert K. Merton and Orlando Patterson?

A. Merton points to the origins of a certain phe-nomenon, while Patterson discusses its effects.

B. The two scholars agree on the existence of aparticular pattern, but are in disagreement as toits causes.

C. Merton’s approach is psychological, while Pat-terson’s is primarily economic.

D. The two scholars are describing two separate,but related, schools of thought.

110. Judging from the passage, the author would probablysupport which of the following types of remedialaction?

A. more stringent enforcement of affirmativeaction policies

B. major changes in the structure of the welfare

program

C. the repeal of affirmative action legislation

D. incentives for manufacturers in urban locations

33

GO ON TO THE NEXT PAGE.

Page 35: Full length 7

Passage VI (Questions 111–116)

Certain contemporary forms of literary criticism drawon modern sociology and political science to understandliterary works. There has been a conservative reaction tothese schools of criticism, accusing them of imposingmodern ideas on old texts. For example, some would con-sider it an implausible claim that the Shakespeare’s TheTempest can be interpreted as a play about “colonialism”and “imperialism”; after all, these terms were not even inuse when Shakespeare wrote the play. These conceptsmust therefore be modern ones, and it is anachronistic tosuppose that Shakespeare had them in mind. Besides, asBen Jonson wrote, Shakespeare “was not of an age, but forall time,” and it trivializes his genius to suppose that he hadin mind the fashionable concerns of any one period. Theconservative reading of The Tempest sees it as a play about“universal” themes like estrangement and reconciliation.

But writers do live in specific societies, and areaffected by the cultures of the times and places in whichthey live. The establishment of colonies, the building ofempires, was an issue of keen concern in England in theearly seventeenth century. It was a matter of national pres-tige, and also a potential source of private wealth. All thegreat powers of Europe were competing for the wealth ofthe East and West Indies. The rich hoped to add to theirfortunes; the poor hoped to begin their lives anew in theNew World. Richard Hakluyt’s Voyages, a series of pub-lished accounts of European explorations in Asia, Africa,and the Americas, was one of the most successful publish-ing ventures of Elizabethan England. Moreover, althoughthe words “colonialism” and “imperialism” had not beencoined yet, the ideas they connote already existed, in thesense that some Europeans perceived ethical problemsrelating to empire-building. The Spanish priest Bartoloméde Las Casas had already condemned the cruelty of theSpanish regime in Mexico and the Caribbean, and theFrench essayist Michel de Montaigne had already com-pared the “Cannibals” favorably with decadent Europeans.

Now let us look again at The Tempest. Here is a playabout a European family ruling a remote island by superiorEuropean technology (magic, learned from books) and theenforced labor of the native population. When anothergroup of Europeans arrives on the island, one of themimagines an ideal commonwealth in terms derived, asscholars have long recognized, from Montaigne’s essayabout the native people of Brazil. The prostrate Calibanreminds Trinculo of a “dead Indian” who might be exhib-ited in England for crowds willing to pay to see an exotic“monster.” And scholars have long recognized that thestory of The Tempest is suggested in part by accounts of theSea Venture, shipwrecked in Bermuda in 1609 on the wayto the Virginia colonies.

With all this in mind, are we really to believe that neitherShakespeare nor anyone who saw the play in London in1611 was reminded of the colonial enterprise that Englandwas then undertaking in America? Who is making theimplausible claim?

111. The author refers to Las Casas and Montaigne inorder to suggest:

A. that they had a greater understanding of sociol-ogy and political science than Shakespeare.

B. that their writings are highly conservative innature.

C. that a person in Shakespeare’s time could havehad concerns about colonialism and imperial-ism.

D. a contrast between Spanish and French litera-ture and English literature in the early seven-teenth century.

112. According to the passage, supporters of the conser-vative reaction to political readings of Shakespearebelieve that:

A. the themes that Shakespeare addressed are time-less.

B. society is a strong influence on the individualartist.

C. colonization is a legitimate way to amasswealth.

D. European technology is superior to other tech-nologies.

113. According to the passage, all of the following aremanifestations of interest in colonization in Shake-speare’s England EXCEPT:

A. the popularity of books about foreign travel.

B. the willingness of poor people to emigrate.

C. the desire to expand English power abroad.

D. the opposition to acts of cruelty by the Spanishin America.

34

GO ON TO THE NEXT PAGE.

5

10

15

20

25

30

35

40

45

50

55

Page 36: Full length 7

114. According to the passage, The Tempest can be saidto:

A. be apolitical and timeless in its appeal.

B. have been written to boost support for coloniza-tion in America.

C. reflect contemporary concerns in a fictionalstory.

D. defend the rights of native peoples.

115. Which of the following best describes the relation-ship of the first paragraph of the passage to the pas-sage as a whole?

A. It makes an assertion that is supported later inthe passage.

B. It briefly compares and contrasts two types ofliterary criticism that are examined in detaillater in the passage.

C. It defines the terms that are used in an argumentlater in the passage.

D. It states the objections to an argument that issupported later in the passage.

116. It can be inferred that the author regards conservativecritics of Shakespeare with:

A. disapproval, because they ignore the influenceof society on an individual.

B. approval, because they emphasize the universal-ity of Shakespeare’s themes.

C. displeasure, because they defend seventeenth-century English colonialism.

D. admiration, because they amass facts aboutShakespeare’s age.

35

GO ON TO THE NEXT PAGE.

Page 37: Full length 7

Passage VII (Questions 117–122)

Labor productivity (defined as output divided by hoursworked) for private business (defined as the entire non-farmbusiness sector, minus the housing industry) is reported tohave been rising steadily at an annual rate of about 1.25percent since 1978. At the same time, productivity in thenon-financial corporate sector is estimated to have beenincreasing at a 1.75 percent annual rate. The non-financialcorporate sector is responsible for about two-thirds of theoutput of all private business, and also for about two-thirdsof all the hours worked. Therefore, these statistics suggestthat output per hour in the remaining third of private busi-ness has not increased for more than two decades.

By far the largest portion of the remaining third iscomposed primarily of sole proprietorships and limitedpartnerships (the non-corporate sector). The remainder,approximately one-seventh, is financial corporations,which present special measurement problems. When focusis placed on the statistics for such typical non-corporateprivate businesses as construction and medical, legal, andbusiness services, the figures point to falling levels of pro-ductivity for many years.

Leaving aside the question of financial corporations, inconsidering these statistics some economists infer that pro-ductivity increases in the corporate sector are primarilydriven by gains in new information and communicationtechnologies, and that these gains may be taking longer todiffuse into the non-corporate sector.

This explanation, however, is wholly at variance withour casual day-by-day experiences. Design times and costshave fallen dramatically as computer modeling has elimi-nated the need, for example, for the large staff of architec-tural specification-drafters previously required for buildingprojects. Medical diagnoses are more thorough, moreaccurate, and far faster, with access to previously unavail-able information. Treatment is accordingly hastened, andhours of procedures eliminated. Furthermore, if the prof-fered explanation were accepted, it would give rise to thequestion: how long does it take for technology-driven effi-ciency gains to occur in the non-corporate sector, and whenthey do take place will economic statistics capture them?

Another explanation that must be considered is thatmethods used in gathering economic data may undercountwhat is produced in the non-corporate sector, consequentlyresulting in miscalculation of its productivity. Since eco-nomic statistics are the basis for significant policy deci-sions and planning activities of government agencies, suchdeficiencies, if they are present in our methods, must beidentified and addressed.

117. It can be inferred from the passage that the portion ofprivate business that consists of corporations is:

A. more than two-thirds.

B. less than those that are non-corporations.

C. less than two-thirds.

D. the same as those that are non-corporations.

118. According to the passage, since 1978 a measure oflabor productivity indicates that it has decreased in:

A. private businesses.

B. corporate businesses.

C. financial corporations.

D. typical non-corporate businesses.

119. According to the passage, some economists believethat productivity gains from new information andcommunications technologies occur:

A. at the same time in all sectors of private busi-ness.

B. earlier in the non-corporate sector and later in

the corporate sector.

C. earlier in the non-financial corporate sector andlater in the non-corporate sector.

D. earlier in the non-financial corporate sector, sec-ond in the non-corporate sector, and last infinancial corporations.

36

GO ON TO THE NEXT PAGE.

5

10

15

20

25

30

35

40

45

Page 38: Full length 7

120. The author refers to computer modeling in architec-tural firms (line 30–34) and availability of medicalinformation (line 34–36) most probably in order to:

A. present evidence that technology is alreadyimproving efficiency in typical non-corporatebusinesses.

B. show how technology diffuses into the non-cor-porate sector.

C. provide examples of improvements in efficiencythat are not readily captured by economic statis-tics.

D. present evidence of how technology used innon-corporate businesses improves the qualityof services consumers use in their daily lives.

121. It can be inferred from the passage that productivityin the non-corporate sector would necessarilyincrease if any of the following occurred EXCEPT:

A. output in the non-corporate sector increased andhours worked in the non-corporate sectordecreased.

B. output in the non-corporate sector increased andhours worked in the non-corporate sector stayedthe same.

C. work hours in all sectors remained the same,output in all private business stayed the same,output of financial corporations stayed thesame, and output in the corporate sectordecreased.

D. output in the non-corporate sector decreasedand hours worked in the non-corporate sectorincreased.

122. The passage suggests that if methods used in gather-ing economic data undercount what is produced inthe non-corporate sector then, if the data were cor-rected, it is most likely that:

A. what is produced in private business as a wholeis undercounted.

B. hours worked in the non-corporate sector areundercounted.

C. what is produced in the non-financial corporatesector is undercounted.

D. hours worked in private business as a whole areundercounted.

37

GO ON TO THE NEXT PAGE.

Page 39: Full length 7

Passage VIII (Questions 123–131)

Until the mid-twentieth century, Baroque art and liter-ature were considered vastly inferior to the art and litera-ture produced during the subsequent Classic period. Fromthe late 1600s through the 1800s, critics and scholars por-trayed Baroque artists and writers as chaotic and disparateproducers of artistic works whose poor style and lack ofaesthetic value were “fixed” by the balanced and orderedaesthetic that evolved and formed the foundation of Clas-sicism. In fact, however, a distinct aesthetic system can beculled by studying Baroque art and literature of the late1500s and early 1600s, and the shared characteristics ofsuch works can be distilled into a consistent view of visualart and literary prose or poetry that is just as valid as theClassic perspective which followed.

For example, visual art produced during the Baroqueperiod was characterized by a lack of boundaries, by anextravagant overflowing of the image beyond its frame,and by a sense of openness and fluidity. Certain paintersincorporated visual tricks into their completed canvases sothat the image perceived would change and shift depend-ing on the relative positions of the spectator and the paint-ing. The Italian painter Giuseppe Archimbaldo, forexample, created a painting that looks like a traditionalstill-life of a bowl of food when viewed from one per-spective, but which changes into the face of a man in astraw-like hat when turned upside-down. Likewise,Archimbaldo’s 1566 painting The Bookseller appears topresent a simple portrait of its title character. However, onclose examination, the portrait in fact consists of a collageof the tools typical of the bookseller’s trade—several vol-umes of books and a variety of ribbon-bookmarks. Byusing such “trompe-l’oeil” techniques, artists diverted theattention of spectators from the concrete reality of theframed image, and spectators were instead forced to re-examine their interpretation of an artistic work and to seekrefuge in the abstract realm of imagination and dreams.

Baroque literature was characterized by a similar lackof boundaries or closure, a constant shifting of characteridentities, a certain extravagance of language, and a senseof openness that prevented a reader from arriving at a fixedand constant interpretation of the story. One typical exam-ple from this period is Honoré d’Urfé’s massive work,l’Astrée. Properly speaking, this narrative work cannot bedescribed as a “novel” since it is far from conforming tothe later conception of the novel formulated during the late1700s and 1800s. Rather, D’Urfé’s story consists of astring of episodes pieced together, with interwoven andcomplicated storylines, which involve humans as well asimmortal deities and mythic figures. Although the workpurported to serve as a model for the behavior of the per-

fect, eternally loyal, and unerringly faithful suitor, muchoccurs during the story that is not intrinsically linked tothis supposed didactic goal. Complications arise fromnumerous disguises and metamorphoses experienced bythe characters. For example, the male suitor Céladonassumes several disguises, eventually cross-dressing andassuming the name Alexis in order to live in close proxim-ity to his true love, Astrée, who has forbidden him to comewithin her sight. He must obey her command, as an idealsuitor, but he realizes that he can safely be near her if he isdisguised as a woman. Eventually, however, this trickeryleads to a loss of fixed identity for Céladon, as well as tocompromising scenes that are interrupted by unrelatedepisodes “just in time” to save Alexis from discovery byAstrée. Written over a period of twenty years (1607–1627),consisting of 5 volumes of 12 books each for a total of5,000 pages, and containing 45 story lines with 200 differ-ent characters, l’Astrée was never completed, and remainsunfinished to this day.

123. According to the passage, art and literature from theBaroque era were considered inferior to art and liter-ature from the Classic era for which of the followingreasons?

A. Artists and writers from the Classic period weremore educated and refined than those of theBaroque period.

B. Artistic works from the Classic era haveremained relevant throughout the centuries butBaroque works were quickly forgotten andneglected.

C. Baroque artists and writers were revolutionarieswhose new ideas stirred up rebellion among thepublic.

D. Literary works of the Baroque period lack a lin-ear plot-line, and Baroque artwork lacks a stan-dard symmetry, both of which were important tothe Classic period.

38

GO ON TO THE NEXT PAGE.

5

10

15

20

25

30

35

40

45

50

55

60

65

Page 40: Full length 7

124. The author of this passage would most likely agreewith which of the following statements?

A. Studying art or literature from neglected erascan reveal valuable creative works that havebeen overlooked due to the vagaries of dominantcritical thought.

B. Baroque artists and writers were just as popularduring their time as were the later Classic artistsand writers.

C. All modern art and literary critics agree that theBaroque era had a coherent aesthetic that can beformulated by studying the creative output ofthe time.

D. The only distinguishing characteristic ofBaroque art and literature was a lack of coher-ence and closure.

125. The primary purpose of this passage is to:

A. defend Baroque art and literature by showingthat the Baroque aesthetic is superior to theClassic aesthetic.

B. provide evidence that the Baroque era had anidentifiable aesthetic that can be traced in bothart and literature of the period.

C. compare and contrast the aesthetic systems ofthe Baroque and Classic eras.

D. advocate additional studies of the Baroque erain order to highlight the artistic merit of theperiod’s art and literature.

126. It can be inferred from the passage that which of thefollowing was most likely a common thematic imageof Baroque literature?

A. mirrors that reflect reality to reveal hiddentruths

B. masks that pass from one character to another toestablish their relationship

C. bodies of water in which reflections mimic thefluid quality of the medium

D. human characters who assume animalistic traits

127. The author describes the specific composition ofl’Astrée primarily in order to:

A. illustrate the length of Baroque narrative fiction.

B. demonstrate that Baroque literature was alwayswritten over a long period of time.

C. show how the vastness of some Baroque literaryworks contributed to their lack of closure.

D. criticize the complexity of lengthy Baroqueprose literature.

128. According to the passage, l’Astrée and The Book-seller share all of the following characteristicsEXCEPT:

A. ambiguous identification of their main charac-ters.

B. overabundance of detail.

C. fluid images which defy a consistent interpreta-tion.

D. lack of a restrictive framework.

129. The passage suggests that which of the following is amodern example of a literary work with a Baroquestructure?

A. A picaresque story in which episodic chapterscreate a unified plot-line.

B. A fantasy novel in which the complicated plot-line jumps between worlds and eras, andinvolves non-human characters.

C, A fairy tale in which the main characters aremagicians and dragons, and whose plot-lineemphasizes a particular moral lesson.

D. A historical novel that explores the relationshipbetween three different generations of an Amer-ican family.

39

GO ON TO THE NEXT PAGE.

Page 41: Full length 7

130. The author most likely mentions Giuseppe Archim-baldo in order to:

A. provide an example that reinforces a hypothesis.

B. describe a specific example that illustrates ageneral principle.

C. present evidence that contradicts a claim withwhich the author disagrees.

D. suggest that a certain artistic aesthetic wasfounded by a particular artist.

131. The passage suggests that which of the following isNOT a potential consequence of the use of disguisesin l’Astrée?

A. cases of mistaken identity which create ironysince the characters know less than the reader

B. confusion on the part of the reader about the truegender of certain characters

C. proof of the loyalty and fidelity of an amorouscharacter

D. humorous scenes in which the disguise leads toan awkward situation

40

GO ON TO THE NEXT PAGE.

Page 42: Full length 7

41

The Test Continues on the Next Page

Page 43: Full length 7

Passage IX (Questions 132–137)

Every known human society prohibits marriagebetween close relatives. This regulation has perplexedanthropologists for decades, because despite its universal-ity, the incest taboo varies—relatives acceptable as mar-riage partners in some societies are unacceptable in others.Since it is both universal and variable, the incest prohibi-tion takes on aspects of both biology and culture. Frenchanthropologist and structuralist Claude Lévi-Straussattempts to account for both of these factors by envision-ing marriage as a kind of circulation or exchange.

Earlier efforts to explain the incest taboo focus oneither its biological or its cultural aspects. Theorists favor-ing biological explanations point to the increased risk ofbirth defects, or postulate an instinctive repugnance forsexual relations with close kin. These explanations ignorethe cultural element of incest prohibitions: if avoidance ofkin as sexual partners were based on a biological impera-tive, there would be no need for cultural rules. Further,worldwide variations in incest taboos seem inconsistentwith a strict biological interpretation. In parts of Africa,marriage is allowed with some first cousins, but not withothers. In the Middle East, cousin marriage is permissible,but the categories of cousins allowed and prohibited are theopposite of those in Africa. And in many other parts of theworld, marriage with first cousins is entirely off-limits.These differences would not be possible were the biologi-cal explanations valid.

Theorists who address the problem in cultural termspropose a learned, rather than biological, lack of sexualinterest among family members. They argue that childhoodfamiliarity with close kin is incompatible with sexualattraction. This explanation fails for two reasons. It doesnot account for some of the global variation in the incesttaboo. In most societies (including North America), rela-tives who do not grow up with a person are nonethelessconsidered off-limits as spouses or lovers. In contrast, anIraqi youngster may grow up with his or her cousin as afrequent playmate, and then marry that cousin in earlyadulthood. The cultural theory also depends on a notion ofmarriage as originating in some variety of romantic love.In many parts of the world, economic and/or political con-cerns, rather than love, constitute the primary motives formarriage.

To resolve these problems, Lévi-Strauss proposes thatmarriage between families or villages constitutes anexchange of wives. He envisions society as a neatlyarranged structure in which, like well-oiled gears, humancommunities ensure their own continuity by establishingalliances with other such groups. Alliances are established

through the circulation of items of incomparable value:human beings themselves, living members of those groups.Daughters of one family or village are sent as wives toanother community, who send their own daughters inreturn. The exchanges ensure settlements’ very survival byextending bonds of socialization and kinship, enablingcommunities to pool resources when harvests are poor, orto unite against common enemies when conflict threatens.The circulation of wives is a cultural phenomenon, but ithas a biological rationale. The model helps explain notonly the prohibitions on marrying in families, but also theenormous success of the human species. Lévi-Strauss thusclears up several of the problems that plague previousexplanations of the incest taboo.

132. The author cites the example of the North Americanincest taboo (lines 34–36) in order to:

A. show that developed nations have the mostrational incest taboos.

B. prove the biological nature of marriage prohibi-tions.

C. cast doubt on cultural theories of marriage pro-hibitions.

D. compare forms of morality worldwide as seen inmarriage practices.

133. Which of the following pieces of evidence, if true,would cast the most doubt on Lévi-Strauss’ theoryexplaining the incest taboo?

A. Scientists find that birth defects frequentlyappear in communities which “marry in” overgenerations, including communities wherecousin-marriage is standard.

B. Missionaries discover an isolated culture inNew Guinea, whose members always marrywithin the group, and who display unusualserenity and health.

C. Historians examining royal families in Europenote that the most successful dynasties are thosewhose daughters regularly marry into royal fam-ilies in neighboring countries.

D. Anthropologists note that the marriage prohibi-tions among a pastoral group in East Africa areidentical to the prohibitions among a villagesociety in highland Southeast Asia; however,other aspects of their culture do not coincide.

42

GO ON TO THE NEXT PAGE.

5

10

15

20

25

30

35

40

45

50

55

60

Page 44: Full length 7

134. The passage implies that manifestations of the incesttaboo are different in different societies because:

A. the level of cultural development varies in dif-ferent parts of the world.

B. in the Middle East, cousin marriage is permissi-ble, but many other societies forbid marriagewith cousins.

C. each continent developed its own taboo due toenvironmental variables.

D. the incest taboo is related to other aspects of thesocial organization in a society.

135. The passage suggests that which aspect of the incesttaboo has made it an unusually difficult phenomenonto explain?

A. It occurs only in humans.B. It is unrelated to other aspects of culture.

C. It can never be violated.

D. It is global, but unique everywhere.

136. The passage suggests that if marriage prohibitionswere purely biological:

A. the incest taboo would probably not exist at all.

B. all birth defects would be eliminated.

C. the incest taboo would have the same meaningin every culture.

D. marriage could result from an exchange ofeither men or women.

137. According to the passage, Lévi-Strauss’ theoryexplains that “the enormous success of the humanspecies” (line 61) was due in part to:

A. ensuring privileges for noble and royal lineages,to preserve societies.

B. the avoidance of birth defects caused by marry-ing within the family.

C. relationships among communities helping themto withstand hardship.

D. humans’ cultural ability to perpetuate the socialsystem in which they are born.

STOP. IF YOU FINISH BEFORE TIME IS CALLED,CHECK YOUR WORK.YOU MAY GO BACK TO ANYQUESTION IN THIS SECTION ONLY.

43

Page 45: Full length 7
Page 46: Full length 7

Writing SampleTime: 60 Minutes

2 Prompts, Separately Timed:30 Minutes Each

DO NOT BEGIN THIS SECTION UNTIL YOU ARE TOLD TO DO SO.

Page 47: Full length 7

46

WRITING SAMPLE

DIRECTIONS: This section is a test of your writing skills. Thesection contains two parts. You will have 30 minutes to completeeach part.

Your responses to the prompts given in the Writing Sample willbe written in the ANSWER DOCUMENT.Your response to Part 1must be written only on the answer sheets marked “1,” and yourresponse to Part 2 must be written only on the answer sheetsmarked “2.” You may work only on Part 1 during the first 30 min-utes of the test and only on Part 2 during the second 30 minutes.If you finish writing on Part 1 before the time is up, you mayreview your work on that part, but do not begin writing on Part 2.If you finish writing on Part 2 before the time is up, you mayreview your work only on Part 2.

Use your time efficiently. Before you begin writing a response,read the assignment carefully and make sure you understandexactly what you are being asked to do. You may use the spacebelow each writing assignment to make notes in planning yourresponses.

Because this is a test of your writing skills, your response to eachpart should be an essay composed of complete sentences andparagraphs, as well organized and clearly written as you canmake it in the allotted time. You may make corrections or addi-tions neatly between the lines of your responses, but do not writein the margins of the answer booklet.

There are six pages in your answer booklet to write yourresponses, three pages for each part of the test. You are notrequired to use all of the pages, but to be sure that you haveenough space for each essay, do not skip lines.

Essays that are illegible cannot be scored. In addition, essaysthat are not written in English will not be scored.

Page 48: Full length 7

47

PART 1

Consider the following statement:

Any business must be concerned with the environmental impact of its actions.

Write a unified essay in which you perform the following tasks. Explain what you think the above statement means. Describea specific situation in which businesses might justifiably not be concerned with the environmental impact of its actions. Dis-cuss what you think determines whether or not a business must be concerned with the environmental impact of its actions.

Page 49: Full length 7

PART 2

Consider the following statement:

A politician must sometimes adopt an unfavorable position for the good of the country.

Write a unified essay in which you perform the following tasks. Explain what you think the above statement means. Describea specific situation in which a politician should not adopt an unfavorable position for the good of the country. Discuss whatyou think determines whether or not a politician must adopt an unfavorable position for the good of the country.

48

Page 50: Full length 7

Biological SciencesTime: 100 MinutesQuestions 138–214

DO NOT BEGIN THIS SECTION UNTIL YOU ARE TOLD TO DO SO.

Page 51: Full length 7

50

BIOLOGICAL SCIENCES

DIRECTIONS: Most of the questions in the BiologicalSciences test are organized into groups, with adescriptive passage preceding each group of ques-tions. Study the passage, then select the single bestanswer to each question in the group. Some of thequestions are not based on a descriptive passage; youmust also select the best answer to these questions. Ifyou are unsure of the best answer, eliminate thechoices that you know are incorrect, then select ananswer from the choices that remain. Indicate yourselection by blackening the corresponding circle onyour answer sheet. A periodic table is provided belowfor your use with the questions.

1

H

1.0

2

He

4.0

3

Li

6.9

4

Be

9.0

5

B

10.8

6

C

12.0

7

N

14.0

8

O

16.0

9

F

19.0

10

Ne

20.2

11

Na

23.0

12

Mg

24.3

13

Al

27.0

14

Si

28.1

15

P

31.0

16

S

32.1

17

Cl

35.5

18

Ar

39.9

19

K

39.1

20

Ca

40.1

21

Sc

45.0

22

Ti

47.9

23

V

50.9

24

Cr

52.0

25

Mn

54.9

26

Fe

55.8

27

Co

58.9

28

Ni

58.7

29

Cu

63.5

30

Zn

65.4

31

Ga

69.7

32

Ge

72.6

33

As

74.9

34

Se

79.0

35

Br

79.9

36

Kr

83.8

37

Rb

85.5

38

Sr

87.6

39

Y

88.9

40

Zr

91.2

41

Nb

92.9

42

Mo

95.9

43

Tc

(98)

44

Ru

101.1

45

Rh

102.9

46

Pd

106.4

47

Ag

107.9

48

Cd

112.4

49

In

114.8

50

Sn

118.7

51

Sb

121.8

52

Te

127.6

53

I

126.9

54

Xe

131.3

55

Cs

132.9

56

Ba

137.3

57

La *

138.9

72

Hf

178.5

73

Ta

180.9

74

W

183.9

75

Re

186.2

76

Os

190.2

77

Ir

192.2

78

Pt

195.1

79

Au

197.0

80

Hg

200.6

81

Tl

204.4

82

Pb

207.2

83

Bi

209.0

84

Po

(209)

85

At

(210)

86

Rn

(222)

87

Fr

(223)

88

Ra

226.0

89

Ac †

227.0

104

Rf

(261)

105

Ha

(262)

106

Unh

(263)

107

Uns

(262)

108

Uno

(265)

109

Une

(267)

*

58

Ce

140.1

59

Pr

140.9

60

Nd

144.2

61

Pm

(145)

62

Sm

150.4

63

Eu

152.0

64

Gd

157.3

65

Tb

158.9

66

Dy

162.5

67

Ho

164.9

68

Er

167.3

69

Tm

168.9

70

Yb

173.0

71

Lu

175.0

90

Th

232.0

91

Pa

(231)

92

U

238.0

93

Np

(237)

94

Pu

(244)

95

Am

(243)

96

Cm

(247)

97

Bk

(247)

98

Cf

(251)

99

Es

(252)

100

Fm

(257)

101

Md

(258)

102

No

(259)

103

Lr

(260)

PERIODIC TABLE OF THE ELEMENTS

Page 52: Full length 7

Passage I (Questions 138–142)

Erythromycin is a potent antibiotic used against arange of bacteria; for example, it is effective againstLegionella pneumophilia, the cause of Legionnaire’s Dis-ease. Erythromycin is produced by the bacteria Strepto-myces erythreaus. Erythromycin consists of a 14-memberring with the sugars mycarose and desosamine attached tothe ring as substituents. Different enzymes perform theassembly of the ring and the attachment of the sugars. Boththe ring and the sugars are necessary for erythromycin tofunction as an effective antibiotic. Two scientists studyingerythromycin proposed different mechanisms for theaction of the drug.

Scientist 1

Erythromycin interferes with synthesis of the peptido-glycan cell wall of bacteria. The cell wall is formed frombuilding blocks of two linked amino sugars joined to anunusual polypeptide containing both L and D amino acids.This cell wall protects the cell from lysis due to osmoticpressure. In the presence of erythromycin, bacteriadevelop thin, weak cell walls that leave them more vul-nerable to environmental damage. The mycarose and des-osamine sugars attached to erythromycin compete withthe sugars used to synthesize the cell wall. This competi-tion inhibits the glycosyl transferases that catalyze thesynthesis of the dissaccharide building blocks used to pro-duce the cell wall.

Scientist 2

Erythromycin interferes with the function of bacterialribosomes. Although eukaryotic ribosomes and prokary-otic ribosomes have similar shapes, eukaryotic ribosomesare larger and more complex than prokaryotic ribosomesand do not appear to be affected by erythromycin. Ery-thromycin interferes with the translocation of the tRNAmolecule and its attached peptide chain; this processoccurs on the large subunit of prokaryotic ribosomes.Translocation of the tRNA-peptide chain is necessary tomake room for the next aminoacyl-tRNA, and if ery-thromycin blocks this process, the growing polypeptidechain is truncated. Since protein synthesis is inhibited, cellgrowth and development are suppressed.

138. Which of the following observations would providethe strongest support for Scientist 2’s hypothesisabout the action of erythromycin?

A. Erythromycin does not inhibit protein synthesisin prokaryotes.

B. An overdose of erythromycin can inhibit mito-chondrial protein synthesis in humans.

C. The bacteria that causes pneumonia can becomeresistant to erythromycin by developing anenzymatic pump to remove erythromycin fromthe cell.

D. Bacteria exposed to erythromycin are morelikely to lyse in hypotonic solutions comparedto unexposed bacteria.

139. Which of the following is the most important differ-ence between the hypotheses of Scientists 1 and 2?

A. Scientist 1’s hypothesis refers to erythromycin’saction in plants while Scientist 2’s hypothesisrefers to erythromycin’s action in bacteria.

B. Scientist 1’s and Scientist 2’s hypotheses referto erythromycin’s action at different sites ofbacterial cells only.

C. Scientist 2’s hypothesis implies that ery-thromycin would be a more powerful drug if itwere true than if Scientist 1’s hypothesis weretrue.

D. Scientist 1’s hypothesis suggests that ery-thromycin would kill the cell, while Scientist 2’shypothesis implies that erythromycin would justslow the cell’s growth.

140. If Scientist 2’s hypothesis is correct, which of the fol-lowing results might be expected if erythromycinwere administered to a plant?

A. The plant would produce novel enzymes todestroy the antibiotic.

B. The plant cells would develop weakened cellwalls.

C. Antibiotic-resistant bacteria would kill theplant.

D. Photosynthesis would be inhibited becausechloroplasts would produce fewer proteins.

51

GO ON TO THE NEXT PAGE.

Page 53: Full length 7

141. Which of the following observations would mostweaken Scientist 1’s hypothesis?

A. Bacteria can live and reproduce in normal envi-ronments without a cell wall.

B. Bacteria often become resistant to antibioticsthat affect the cell wall by modifying the struc-ture of the cell wall.

C. The enzymes that produce the bacterial cell wallare not produced by the ribosomes.

D. Erythromycin has no effect on viruses.

142. Which of the following is true about the genes for theenzymes that synthesize erythromycin?

A. They could only have come from viruses.

B. They contain introns that must be spliced outbefore transcription and translation can occur.

C. They are found on a circular chromosome.

D. They are found near the telomere of the bacte-rial chromosome.

Passage II (Questions 143–148)

A student was given four or five organic compounds ina vial. The compounds are naphthalene, 4-methylaniline,p-cresol, 2-methylbenzoic acid, and possibly one moreunknown compound. The structures of the four knowncompounds are shown below.

The student performed the following extractionsequence.

mixture

Step 1(dimethyl ether/HCO3

-)

aqueous

aqueous

aqueous

organic

organic

organic

(+ HCl)

Compound A

Step 2(+ NaOH)

p-cresol

Step 3(+ HCl)

(+ NaOH)Step 4(anhydrous CaCl2,evaporate to dryness)

Compound B Compound C

?

Naphthalene

COOH

CH3

2-Methylbenzoic Acid

4-Methylaniline

OHNH2

CH3 CH3

p-cresol

52

GO ON TO THE NEXT PAGE.

Page 54: Full length 7

53

GO ON TO THE NEXT PAGE.

143. In Step 2, after the separation of the aqueous andorganic layers, what was most likely added to theaqueous layer to extract p-cresol?

A. NaOH

B. HCl

C. HCO3–

D. anhydrous CaCl2

144. In the aqueous phase after Step 1 but before the addi-tion of HCl, the predominant form of Compound A is:

A. an amine salt.

B. an amine.

C. a phenoxide ion.

D. a carboxylate ion.

145. In which step would the student need to take specialcare to properly avoid pressure buildup in the separa-tory funnel?

A. Step 1

B. Step 2

C. Step 3

D. Step 4

146. Which of the following is the most appropriate tech-nique to determine if Compound C contains theunknown fifth compound?

A. NMR

B. IR

C. TLC

D. Mass spectrometry

147. If the fifth, unknown compound were benzylamine,from which step would it be isolated?

A. Step 1

B. Step 2

C. Step 3

D. Step 4

148. In which chemical shift region do the four unknowncompounds all have signals in their proton NMRspectra?

A. 0.5-0.9 ppm

B. 2-3 ppm

C. 4.5-6 ppm

D. 6-8.5 ppm

Passage III (Questions 149–155)

Hormonal action associated with the endocrine systemprovides the body with mechanisms with which to controlphysiological functioning and to maintain homeostasis. Ofparamount importance to this internal regulation are theadrenal glands. Located atop the superior border of thekidneys, each adrenal gland consists of an outer cortex andinner medulla, which function as separate glands.

The cortex is the more active component of the gland;it is divided into three zones, each having a unique role inhormone production. The superficial layer, or zonaglomerulosa, produces mineralocorticoids, such as aldos-terone. Such corticoids directly regulate ion balance and,therefore, indirectly control blood volume. Specifically,aldosterone allows for Na+ reabsorption from the nephronsby active transport. Acting on certain cells lining nephrontubules, aldosterone turns on active sodium ion reabsorp-tion so that sodium passes from the filtrate back into thebloodstream. This, in turn, brings water into the blood,increasing blood volume and blood pressure.

The middle layer of the cortex, or zona fasiculata,secretes glucocorticoids such as cortisol, a hormonederived from cholesterol. Cortisol and other glucocorti-coids promote the synthesis of glucose from non-carbohy-drate sources within the body and make glucose availableas fuel. However, they also have a tendency to suppress theinflammatory response as well as other aspects of theimmune system. Exposure to stress stimulates the anteriorpituitary to produce adrenocorticotropic hormone(ACTH), which stimulates release of hormones from theadrenal cortex. This may explain why people under stressmay be more susceptible to disease. The deep layer of thecortex, or zona reticularis, is believed to secrete androgensin both males and females. Some evidence indicates thatthis layer may also secrete cortisol.

The body, or medulla, of the adrenal gland is derivedfrom the embryonic neural crest ectoderm, the same tissuethat produces the sympathetic nervous system. As withother tissues, its embryonic development influences muchof its functioning. For example, the medulla is stimulatedby nerve fibers rather than by hormones like other areas ofthe adrenal glands.

Page 55: Full length 7

149. A patient with Cushing’s syndrome, caused by anoversecretion of ACTH, is most likely to exhibit:

A. hypoglycemia.

B. hypertension.

C. increased resistance to disease.

D. uncontrolled muscle growth.

150. The action of aldosterone in the nephron:

A. occurs mainly at the distal convoluted tubuleand requires ATP.

B. occurs mainly at the proximal convoluted tubuleand does not require ATP.

C. occurs mainly at the descending loop of Henleand may require ATP depending on concentra-tion gradients.

D. is inhibited by low blood pressure.

151. With respect to regulating water reabsorption, aldos-terone has the same effect as:

A. increased vasopressin secretion.

B. decreased antidiuretic hormone (ADH) secre-tion.

C. decreased blood osmolarity.

D. increased cholecystokinin (CCK) secretion.

152. A tumor of the adrenal medulla, or pheochromocy-toma, could result in all of the following EXCEPT:

A. increased heart rate.

B. stimulation of salivation.

C. pupil dilation.

D. decreased blood flow to the stomach.

153. In males, the zona reticularis functions most simi-larly to the:

A. parathyroid.

B. zona fasiculata.

C. kidneys.

D. testes.

154. In order to be classified as a hormone, a chemicalmust be:

A. derived from cholesterol.

B. secreted into a body fluid.

C. secreted into a duct.

D. derived from an amino acid.

155. Which of the following hormones uses cAMP as asecond messenger?

A. Cortisol

B. Testosterone

C. Epinephrine

D. Progesterone

54

GO ON TO THE NEXT PAGE.

Page 56: Full length 7

Passage IV (Questions 156–161)

The molecular technique of DNA probing using gelelectrophoresis is a powerful tool in genetic screening fordiseases. Molecular “probes” are radioactive pieces ofDNA that are used to identify the presence of specificnucleotide sequences in samples of DNA. The radioactiveprobes will hybridize, or match up with, any complemen-tary sequences that exist in the sample.

In order to screen for a particular disease allele, it isfirst necessary to know a significant portion of the DNAsequence for the gene in question, if not the wholesequence of that gene. DNA fragments are loaded into agel to which electrodes are attached at both ends. Whenthe current is turned on, the DNA fragments are drawntoward the anode, since the phosphate groups on nucleicacids give DNA a negative charge. Larger DNA fragmentstravel more slowly than smaller DNA fragments throughthe gel. When the gel is done running, a filter is placed onthe gel, and the DNA that has migrated to different placesalong the gel is transferred onto the filter. The filter is thenplaced in a solution containing the molecular probe con-sisting of a set sequence of DNA bases. After hybridiza-tion, a photographic film is placed above the filter. If aradioactive probe has attached to a complementary frag-ment of DNA on the filter, it will expose the film directlyabove where the probe has hybridized. This procedure cantest for the presence of specific genes and can therefore beused to screen individuals for the presence of disease-causing alleles.

In the pedigree shown, individual I-1 is affected with aparticular autosomal recessive disorder and individual I-2is a carrier for the disorder. The disease allele differs fromthe “normal” allele by a 500 base nucleotide addition. Thegenotypes of the rest of the individuals have not yet beendetermined. However, DNA samples from the individualshave been taken and probed with a DNA sequence match-ing a large part of the recessive, disease-causing allele. Theresults are shown below the pedigree, underneath the cor-responding individuals (Figure 1).

Figure 1

156. Which child in generation II has a 50% chance ofpassing along the disease allele to his/her children?

A. Child 1

B. Child 3

C. Child 5

D. All of the children in generation II have thesame 50% chance of passing on the gene.

157. What is the chance that individuals II-5 and II-6 willhave two children in a row that carry the disease-causing allele?

A. 0%

B. 25%

C. 75%

D. 100%

158. If 96% of the individuals in a population have at leastone copy of the dominant, “normal” allele as part oftheir genotype, what percentage of the populationwill be carriers of the disorder referred to in the pas-sage?

A. 4%

B. 16%

C. 32%

D. 80%

?II

I

III

1 2 3 4 5 6

21

1 2

3

?

?

? ? ?

? ?

?

??

Affected

Carrier

Unknown

55

GO ON TO THE NEXT PAGE.

Page 57: Full length 7

159. Individuals I-1 and I-2, along with the married cou-ple II-5 and II-6, decide to move to a deserted islandand start a community. This community would likelyhave a higher frequency of individuals who eitherhave the disorder or carry the allele for the disorderthan most other areas of the world. This would be anexample of:

A. natural selection.

B. a founder effect.

C. a bottleneck effect.

D. genetic drift.

160. If doctors had wanted to double-check the results ofa positive screen for this disorder in a particular indi-vidual by analyzing proteins found in the individual’sbody, they might use which of the following tools?

A. Radioactive sulfur

B. Radioactive nitrogen

C. Probes for mRNA

D. Probes for histone proteins

161. Which of the following observations would lead ageneticist to suspect that an inherited disorder of cellmetabolism is due to a defective mitochondrial gene?

A. All daughters of a male who has the trait willalso have the trait.

B. There is no male to male transmission of thetrait.

C. The trait is passed down only from mothers andnot fathers.

D. The trait often skips generations.

162. The mRNA sequence shown below is transcribedfrom which of the following DNA fragments?

5'-UCUCCAG-3'

A. 5'-AGAGGUC-3'

B. 5'-CUGGAGA-3'

C. 5'-AGAGGTC-3'

D. 5'-CTGGAGA-3'

163. The 1H-NMR spectrum of 2-chloropropane containswhich of the following signals?

A. A septet and two doublets

B. A septet and a doublet

C. Two quartets and a doublet

D. Two quartets and a triplet

164. Which of the following structures is derived from thesame germ layer as the lungs?

A. Stomach

B. Skeletal muscles

C. Brain

D. Kidney

165. If the diploid number of a female organism is 36,then the number of chromosomes in each egg cell is:

A. 9

B. 18

C. 36

D. 72

56

GO ON TO THE NEXT PAGE.

Questions 162 through 165 are NOT based on adescriptive passage.

Page 58: Full length 7

Passage V (Questions 166–171)

Traditionally, oxidation of alcohols is accomplishedvia the use of chromium-containing reagents such as PCCor PDC or Jones reagent. Oxidation of a primary alcoholresults in the formation of either an aldehyde or a car-boxylic acid, depending on the reaction conditions. Forexample, in nonaqueous conditions, oxidation by PCC orPDC stops at the aldehyde stage. However, addition ofwater immediately oxidizes the aldehyde to the carboxylicacid. Secondary alcohols are oxidized to yield ketones.

A new, inexpensive and environmentally consciousreaction for the catalytic oxidation of alcohol in water hasbeen developed. The reaction involves the use of a water-soluble catalyst containing palladium.

A scientist carried out the oxidation of various alcoholsusing this method and determined the percent yield foreach. The results are given in Figure 1. For each reaction,equal volumes of the alcohol and the aqueous catalystsolution were reacted, creating a two-phase system. Thecatalyst, palladium (II) bathophenanthroline, Pd-(phenS),complexes with O2, which is bubbled through the aqueoussolution. Through partitioning, the two-phase systemallowed contact between the alcohol molecules, oxygen,and the water-soluble catalyst. The rate of reactionincreased with the solubility of the alcohol in water. Theuse of TEMPO (2, 2, 6, 6-tetramethylpiperidinyl-1-oxy)stopped the reactions in vessels C and E at the aldehydestage. After each reaction the product was simply decantedand the catalyst recycled. The purity and percent yield foreach reaction was determined by gas chromatography.

Figure 1 Percent yield of oxidation of various alcohols

166. Which one of the following reactants would not be ableto undergo oxidation with the Pd-(phenS) catalyst?

A. 2,3-Dimethyl-2-hexanol

B. 2,3-Dimethyl-1-pentanol

C. 3,3-Dimethyl-2-octanol

D. 2,2-Dimethylheptanal

OH O

Vessel A (90% yield):

bp (°C): 118-119 100-110

Pd-(phenS)

OH O

Vessel B (90% yield):

bp (°C): 136 127

Pd-(phenS)

OH

OH

O

Vessel C (90% yield):

bp (°C): 137 103

Pd-(phenS)

TEMPO

TEMPO

OH O

Vessel C (80% yield):

bp (°C): 156 202

Pd-(phenS)

OH O

Vessel D (93% yield):

bp (°C): 205 179

Pd-(phenS)

57

GO ON TO THE NEXT PAGE.

Page 59: Full length 7

58

GO ON TO THE NEXT PAGE.

167. The rate of oxidation using Pd-(phenS) in a two-phase system would be fastest with which of the fol-lowing reagents?

A. 1-Pentanol

B. 3-Methyl-2-nonanol

C. 1-Decanol

D. 3-Methyl-2-octanol

168. Which of the following compounds has the highestboiling point?

A. 3-Pentanone

B. 3-Hexanone

C. Heptanoic acid

D. Octanoic acid

169. Which of the following is the best technique to dis-tinguish between the products of Vessels A and B?

A. IR spectroscopy

B. UV-vis spectroscopy

C. Thin-layer chromatography

D. Gas chromatography

170. What is the product of the reaction between 3-hex-anol and PCC in nonaqueous conditions?

A. Hexanal

B. 3-Hexanone

C. Butanoic acid

D. Hexanoic acid

171. Which of the following reaction schemes would leadto the formation of butanal?

I. Oxidation of 1-butanol with PCC or PDCin aqueous conditions

II. Oxidation of 1-butanol with PCC or PDCin nonaqueous conditions

III. Reaction of 1-butanol with Pd-(phenS) cat-alyst in the absence of TEMPO

IV. Reaction of 1-butanol with Pd-(phenS) cat-alyst in the presence of TEMPO

A. I and III only

B. I and IV only

C. II and III only

D. II and IV only

Passage VI (Questions 172–178)

Fetal circulation differs from adult circulation becausethe fetus obtains O2 and nutrients and also eliminates itsCO2 and wastes by diffusion through the placenta. The pla-centa allows exchange of materials between the maternaland the fetal circulatory systems without direct mixing ofblood. In order for the fetus to obtain a sufficient amountof O2 from the mother, fetal hemoglobin has long beenpostulated to have a higher affinity for O2 than maternalhemoglobin. An experiment was conducted to determinethe percent saturation of hemoglobin at varying partialpressures of oxygen for both maternal and fetalhemoglobin. The results of the experiment are shown inFigure 1.

Figure 1 Percent saturation of maternal and fetal Hb

In addition to binding oxygen, human hemoglobin canalso bind other co-factors that affect hemoglobin’s affinityfor oxygen. One such cofactor is D-2,3-bisphosphoglycer-ate (BPG), which is found in the same concentration inadult and fetal erythrocytes. Investigators measured theaffinity of maternal and fetal hemoglobin for BPG by vary-ing the concentration of BPG and monitoring the percentof hemoglobin bound to BPG. The results of this experi-ment are shown in Figure 2.

Figure 2 Percent of maternal and fetal Hb bound to BPG

0

20

40

60

80

100

0 0.5 1 1.5 2

[BPG] (mM)

Per

cent

of

hem

oglo

bin

boun

d to

B

PG

Maternal Hb

Fetal Hb

0

20

40

60

80

100

0 15 30 45 60 75 90Partial pressure of oxygen (mm Hg)

(at room temperature)

Per

cent

sat

urat

ion

of

hem

oglo

bin

Maternal Hb

Fetal Hb

Page 60: Full length 7

172. Myoglobin and hemoglobin are both oxygen-bindingproteins in vertebrates. However, their oxygen disso-ciation curves are very different as illustrated in thefigure below.

Which of the following statements best describes thereason for the difference?

A. Hemoglobin binds oxygen with higher affinityand has cooperative interaction between sub-units.

B. Myoglobin binds oxygen with higher affinityand has cooperative interaction between sub-units.

C. Hemoglobin binds oxygen with lower affinityand has cooperative interaction between sub-units.

D. Myoglobin binds oxygen with lower affinityand has no cooperative interaction between sub-units.

173. Adaptation to high altitudes involves a series of com-plex responses, one of which leads to a rapid increasein the erythrocyte BPG concentration. As a conse-quence of this response there is:

A. an increase in hemoglobin’s affinity for oxygento facilitate uptake of more O2 under reducedPO2

conditions.

B. a decrease in hemoglobin’s affinity for oxygento facilitate release of oxygen to the tissues.

C. an increase of hemoglobin’s affinity for oxygenwhich enhances its ability to unload CO2 andbind O2 in the lungs.

D. a decrease in hemoglobin’s affinity for oxygenwhich enhances its ability to release O2 in thelungs.

174. If the partial pressure of oxygen in a resting humanadult is approximately 40 mm Hg in systemic veinsand 100 mm Hg in systemic arteries, then which offollowing represents the percent of hemoglobin thatis not saturated with oxygen in systemic veins andarteries respectively?

A. 25% and 0%

B. 40% and 15%

C. 60% and 25%

D. 75% and 100%

175. All of the following cause the oxygen-hemoglobindissociation curve to shift to the right EXCEPT:

A. increased pH.

B. increased partial pressure of carbon dioxide.

C. increased body temperature.

D. increased concentration of BPG inside erythro-cytes.

176. After fetal blood picks up O2 at the placenta it trav-els to the liver via a single umbilical vein. The bloodis then diverted from the liver into the inferior venacava via the:

A. ductus arteriosus.

B. foramen ovale.

C. ductus venosus.

D. pulmonary artery.

177. Which of the following might enable one to changematernal hemoglobin’s affinity for oxygen to moreclosely resemble fetal hemoglobin’s affinity foroxygen?

A. Lowering [pH].

B. Lowering BPG concentration inside erythro-cytes.

C. Raising the temperature.

D. Raising CO2 concentration.

0

20

40

60

80

100

0 15 30 45 60 75 90

Partial pressure of oxygen (mm Hg)

Per

cent

sat

urat

ion

Hemoglobin

Myoglobin

59

GO ON TO THE NEXT PAGE.

Page 61: Full length 7

178. Carbon monoxide (CO) is a colorless, odorless gasthat can bind to the heme group of hemoglobin inplace of O2 with an affinity over 200 times greaterthan that between O2 and hemoglobin. What is theeffect of small amounts of CO on the oxygen-hemoglobin dissociation curve?

A. The maximum oxygen carrying capacity ofhemoglobin will be lower.

B. The oxygen-hemoglobin dissociation curve willshift to the left.

C. The oxygen-hemoglobin dissociation curve willremain unchanged.

D. The oxygen-hemoglobin dissociation curve willshift upward.

Passage VII (Questions 179–183)

While only a small fraction of food is digested in thestomach, gastric functions play an integral role in the pro-cessing of food. These functions are conveniently dividedinto three distinct phases: cephalic, gastric, and, intestinal.During the cephalic phase, the vagus nerve stimulates thesecretion of pepsinogen, histamine, and gastrin from cellslining the stomach. The secreted gastrin enters the sys-temic circulation, stimulates the release of histamine, andleads to a decrease in gastric pH. The cephalic phase, asthe name implies, originates from brain centers as thearrival of foodstuffs in the stomach is anticipated.

Once food enters the stomach, the gastric phase domi-nates. Partially digested proteins stimulate the secretion ofpepsinogen and gastrin from chief cells and G cells,respectively. Stomach acid concentration is regulated by anegative feedback loop where decreasing pH is correlatedwith a reduction in gastrin secretion and vice-versa. Lowergastrin levels, thus, reduce the amount of HCl producedfrom parietal cells. Proteins in the stomach help to bufferthe acid, preventing a rapid fall in pH. In this way, the pres-ence of polypeptides and proteins inhibits the negativefeedback loop just described, allowing the amount of acidsecreted to be tailored to the amount of protein ingested.

The final phase of gastric activity, the intestinal phase,includes the complete inhibition of gastric activity. Thearrival of chyme into the beginning of the small intestineproduces a neural reflex that inhibits gastric secretion.Enzymes such as cholecystikinin (CCK), secretin, and gas-trin are connected to the opening and closing of the pyloricsphincter, a narrow band of muscle between the end of thestomach and the duodenum, which allows precise amountsof acid chyme to be delivered to the intestines for regulateddigestion.

A physiologist wishes to examine enzyme deficienciesin two different families. An enzyme-deficient subjectfrom each of the families is first given a protein-rich meal,after which the physiologist notes abnormalities as shownin Table 1.

Patient 1 – Gastric pH = 4.5 (abnormally high)– Large amounts of undigested protein

remain in intestines

Patient 2 – Gastric pH = 1.1 (abnormally low)– Normal protein digestion

Table 1

60

GO ON TO THE NEXT PAGE.

Page 62: Full length 7

The patients are then fed a high-fat meal. The physiol-ogist runs the same tests and reports his results in Table 2.

Patient 1 – Gastric pH = 2.3 (within normal range)– Stomach secretion reduced – Normal fat digestion

Patient 2 – Gastric pH = 2.3 (within normal range)– Stomach secretion reduced – Large amounts of undigested fat remain in

intestines

Table 2

179. The study suggests that a high concentration of bilesalts in the small intestine should correlate withwhich of the following conditions in the stomach?

A. Lower gastrin secretion

B. Decreased pH

C. Increased chief cell activity

D. High pepsinogen levels

180. Patient 1 and Patient 2 are similar in that both showabnormalities:

A. during protein digestion.

B. during fat digestion.

C. during carbohydrate digestion.

D. in regulation of gastric pH.

181. The experimental results indicate that Patient 2 hastwo enzyme deficiencies. These deficiencies proba-bly involve enzymes found in Patient 2’s:

I. stomach.

II. large intestine.

III. small intestine.

A. I only

B. III only

C. I and III

D. II and III

182. Which of the following might stimulate CCKrelease?

A. Presence of chyme in the small intestine

B. A high rate of pepsinogen production

C. A low concentration of fatty acids in ingestedfood

D. Pyloric sphincter contraction

183. Gastrin interacts with parietal cells in a way that isconsidered negative feedback because:

A. a decrease in acid production leads to a decreasein gastrin release.

B. a decrease in pH leads to a decrease in gastrinrelease.

C. an increase in pH leads to a decrease in gastrinrelease.

D. an increase in acid production leads to anincrease in gastrin release.

61

GO ON TO THE NEXT PAGE.

Page 63: Full length 7

Passage VIII (Questions 184–188)

Aldohexoses have four stereogenic centers with a totalof 16 possible stereoisomers. Eight of the stereoisomersare shown in Figure 1.

Figure 1 Stereoisomers of aldohexoses

The aldohexoses shown are all in the D-configuration,in which the hydroxy group of the highest-numbered stere-ogenic center is drawn to the right. The remaining eightpossible stereoisomers are the enantiomers of these struc-tures.

In the late 1800s Emil Fischer conducted a series ofexperiments to deduce the structure of (+)-glucose. (+)-Glucose is expected to be one of the 16 possible structures,but Fischer (arbitrarily) decided to limit his possibilities toonly those structures with the D-configuration, with theunderstanding that the actual structure may be the enan-tiomer.

Some of the reactions that monosaccharides canundergo are described below, together with the results Fis-cher obtained when he carried them out on (+)-glucose andrelated compounds.

Nitric acid oxidationNitric acid oxidizes the aldehyde and primary alcohol

functional groups of monosaccharides to the carboxylgroup. The resulting compound is a dicarboxylic acidknown as an aldaric acid. When Fischer carried out theoxidation of (+)-glucose with nitric acid, the resultingaldaric acid is optically active.

Kiliani-Fischer synthesisThe Kiliani-Fischer synthesis reaction lengthens the

chain of an aldose by one carbon at the aldehyde group.This creates a new stereogenic center in the monosaccha-ride. Both configurations would be obtained in the productmixture. When Fischer carried out the reaction on (−)-ara-binose, an aldopentose, he obtained a mixture of (+)-glu-cose and (+)-mannose.

184. Which compounds shown in Figure 1 would NOT beoptically active after oxidation by nitric acid?

A. Compounds I and IV only

B. Compounds I and VII only

C. Compounds IV and VI only

D. Compounds IV and VII only

185. The structure of L-gulose is shown below. Whichcompound shown in Figure 1 is D-gulose?

A. Compound III

B. Compound IV

C. Compound V

D. Compound VI

HO

CHO

H

H HO

H OH

H HO

CH2OH

CHO

H OH

H OH

H OH

H OH

CH2OH

CHO

H HO

H OH

H OH

H OH

CH2OH

C HO

H OH

H HO

H OH

H OH

C H2OH

CHO

H HO

H HO

H OH

H OH

CH2OH

I II III

CHO

H OH

H OH

H HO

H OH

CH2OH

CHO

H HO

H OH

H HO

H OH

CH2OH

IV V VI

CHO

H OH

HO

H

H

HO

H OH

CH2OH

V II

CHO

HO

HO

H

H

H

HO

H OH

CH2OH

VIII

62

GO ON TO THE NEXT PAGE.

Page 64: Full length 7

186. Which of the following compounds is a ketohexose?

187. Based on the results of the Kiliani-Fischer synthesisreaction, which of the following pairs of compoundscould be (+)-glucose and (+)-mannose, not necessar-ily respectively?

A. Compounds II and III

B. Compounds III and IV

C. Compounds V and VII

D. Compounds VI and VII

188. Which of the following is a correct representation ofthe cyclic form of compound VII?

A.

OH O

OH

OH

CH2OH

OH

B.

OH

OOHOH

CH2OH

OH

C.

OH OOH

OH

CH2OH

OH

D.

OH OOHOH

CH2OH

OH

A. C.H

O

H OH

H OH

H HO

OHH

CH2OH

H HO

O

CH2OH

OHH

OHH

OHH

CH2OH

B. D.

H OH

O

CH2OH

H HO

CH2OH

CH2OH

O

H OH

HHO

OHH

CH2OH

63

GO ON TO THE NEXT PAGE.

Page 65: Full length 7

64

GO ON TO THE NEXT PAGE.

189. The process by which nerve cells transport positiveions from their cytoplasm into the extracellular fluidin order to return their membrane potential from +50mV to –70 mV is known as:

A. depolarization.

B. hyperpolarization.

C. repolarization.

D. polarization.

190. Which of the following statements regarding Okasakifragments is true?

A. Okasaki fragments remain on the lagging DNAstrand during DNA replication because of dis-continuous DNA polymerase action.

B. Okasaki fragments are the equivalent of exons,spliced out of mRNA and rejoined in order toleave the nucleus.

C. Okasaki fragments result from DNA damagedue to UV light or other carcinogens and arecarefully spliced out.

D. Okasaki fragments are built off a DNA primerby DNA polymerase acting away from the ori-gin of replication.

191. An organism is a virus rather than a bacterium if it:

A. can synthesize proteins.

B. can metabolize nutrient molecules.

C. contains membrane-bound organelles.

D. consists of nucleic acid in a protein coat.

192. How many constitutional isomers exist for the for-mula C5H12?

A. 3

B. 4

C. 5

D. 6

193. Which of the following families of compounds willlead to the decoloration of bromine in carbon tetra-chloride?

A. Alkenes

B. Alcohols

C. Ethers

D. Ketones

Passage IX (Questions 194–198)

Esters and amides can undergo hydrolysis in bothacidic and basic conditions. In living systems, the processcan also be catalyzed by enzymes.

Acid-catalyzed hydrolysisIn the acid-catalyzed hydrolysis of esters or amides,

the acid protonates the carbonyl oxygen, facilitating thenucleophilic attack by water on the compound. A tetrahe-dral addition intermediate is formed. The leaving group isreleased after the oxygen or nitrogen is protonated. Depro-tonation of the carbonyl oxygen regenerates the acid cata-lyst, and the product is a carboxylic acid.

Base-promoted hydrolysisIn base-promoted hydrolysis, a hydroxide ion, rather

than a water molecule, is the nucleophile that attacks thecarbonyl compound. The leaving group is an alkoxide ion,or, in the case of amide hydrolysis, an amide ion thatabstracts a proton from water in a concerted step. Theresulting carboxylic acid rapidly deprotonates to yield thecarboxylate ion.

Enzyme-catalyzed hydrolysisSerine proteases are enzymes that catalyze the hydrol-

ysis of peptide bonds. They also allow hydrolysis of esterlinkages. Figure 1 shows the active site of a serine proteasecatalyzing the peptide bond hydrolysis.

Questions 189 through 193 are NOT based on adescriptive passage.

Page 66: Full length 7

Figure 1 Amide hydrolysis catalyzed by a serine protease

C O

O

HN

N

CH2

Asp 102

His 57

H2C

OH

Ser 195

RR'HN

O

C O

O

HN

N

CH2

Asp 102

His 57

CH2

Ser 195

H

R'HN C

O

R

O

Step 1

Step 2 C O

O

HN

N

CH2

Asp 102

His 57

H2C

Ser 195

H

NHR'

OCO

R

Step 3

H2O, − NH2R'

C O

O

HN

N

CH2

Asp 102

His 57

H2C

Ser 195

H

OCO

R

OH

Step 4C O

O

HN

N

CH2

Asp 102

His 57

H2C

Ser 195

O

CH

OH R

O

Step 5 C O

O

HN

N

CH2

Asp 102

His 57

H2C

OH

Ser 195

C O

HO

R

65

Page 67: Full length 7

194. Which of the following statements about thereversibility of ester hydrolysis is accurate?

A. Ester hydrolysis is reversible in acidic condi-tions but irreversible in basic conditions.

B. Ester hydrolysis is reversible in basic conditionsbut irreversible in acidic conditions.

C. Ester hydrolysis is reversible in both acidic andbasic conditions.

D. Ester hydrolysis is irreversible in both acidicand basic conditions.

195. If the following isotopically labeled ester undergoeshydrolysis catalyzed by a serine protease, where willthe isotopic label end up?

A. In the carboxylic acid molecule

B. In an alcohol molecule

C. In a water molecule

D. At serine 195

196. What are the products of the base-promoted hydroly-sis of N-propylbutanamide?

A. Propanoic acid and 1-butanamine

B. Propanoate ion and 1-butanamine

C. Butanoic acid and 1-propanamine

D. Butanoate ion and 1-propanamine

197. Which of the following CANNOT be partiallyresponsible for the fact that amide hydrolysis is irre-versible in both acidic and basic conditions?

A. At low pH, the amine product becomes proto-nated.

B. At low pH, the nitrogen atom in the tetrahedralintermediate favors being protonated.

C. At high pH, amide hydrolysis involves the elim-ination of the strongly basic NH2

− as leavinggroup.

D. At high pH, the hydrolysis product is the car-boxylate ion.

198. In Figure 1, in which step is an acyl-enzyme inter-mediate formed?

A. Step 1

B. Step 2

C. Step 3

D. Step 4

O

O18

66

GO ON TO THE NEXT PAGE.

Page 68: Full length 7

67

GO ON TO THE NEXT PAGE.

Passage X (Questions 199–204)

During development, the survival and differentiation ofcells depends on extracellular signals that are produced byother cells. Many of these signals are peptide growth factorsthat control diverse cellular functions by activating specificsignaling pathways. Insulin-like growth factor (IGF) playsan important role in the formation and maintenance of skele-tal muscle. Mice that lack either IGF itself or IGF receptorsdie shortly after birth because they fail to develop adequatemuscle mass. Furthermore, skeletal muscle cell precursors(myoblasts) in tissue culture fail to differentiate into skele-tal muscle fibers unless IGF is added to the culture.

In an attempt to investigate the signal transductionpathway involved in the IGF-mediated muscle cell survivaland differentiation, researchers isolated three mutantmyoblast cell lines. All three mutant cell lines had normalIGF receptors but failed to survive and differentiate intomature muscle cells in response to IGF. The first mutantcell line contained a mutation in a gene coding for a pro-tein called cyclin-dependent kinase inhibitor, p21. The sec-ond mutant cell line had a mutation in the transcriptionalactivator MyoD. The third cell had a nonfunctional form ofthe enzyme phosphatidylinositol-3-kinase (PI3-kinase).

MyoD, PI3-kinase and p21 act in the IGF-mediatedpathway crucial to muscle cell survival and differentiation.The three proteins become activated in sequential mannerafter IGF binds to receptors on the plasma membrane.Inside the cell, the activation signal proceeds in one direc-tion where each stimulated component activates the nextcomponent of the pathway but has little to no effect on thepreceding component. The initial IGF signal is therebytransmitted until a cellular response is initiated.

To determine the order in which these three proteinsact in the IGF-mediated signal transduction pathway,researchers artificially activated each of the three proteinsindividually and determined the response of the remainingtwo proteins. The results of the experiments are plotted inFigures 1–3.

Figure 1 Activity of PI3-kinase and p21 upon activationof MyoD

Figure 2 Activity of MyoD and p21 upon activation ofPI3-kinase

Figure 3 Activity of PI3-kinase and MyoD upon activa-tion of p21

199. Which of the following may delay maturation ofmyoblasts into muscle cells?

A. Activation of phosphatidylinositol 3-kinase(PI3-kinase)

B. Administration of insulin-like growth factor(IGF) to growing cells

C. Mutations in the gene coding for IGF receptors

D. Administration of MyoD in the presence of p21

200. Which of the following is most likely to have a ther-apeutic effect on mice that die because they lack IGFreceptors?

A. Administering a drug that stimulates PI3-kinase

B. Administering a drug that inhibits p21

C. Administering a drug that mimics IGF

D. Administering a drug that increases PI3-kinaseactivity upon activation of MyoD

0

5

10

15

20

25

0 5 10 15 20 25

Time (hours)R

elat

ive

pro

tein

act

ivit

y

PI3-kinase

MyoD

0

5

10

15

20

25

0 5 10 15 20 25

Time (hours))

Rel

ativ

e pr

otei

n ac

tivi

ty

p21

MyoD

0

5

10

15

20

25

0 5 10 15 20 25

Time (hours)

Rel

ativ

e pr

otei

n ac

tivi

ty

p21PI3-Kinase

Page 69: Full length 7

201. According to the figures presented in the passage, themost probable sequence of events that occurs inmyoblasts after IGF binds its receptor involves stim-ulation of:

A. MyoD followed by activation of p21 and PI3-kinase.

B. PI3-kinase followed by activation of MyoD andp21.

C. p21 followed by activation of MyoD and PI3-kinase.

D. MyoD followed by inhibition of p21 and PI3-kinase.

202. Which of the following might be used to promotesurvival and differentiation in the mutant cell linecontaining a mutation in the gene coding for MyoD?

A. Activation of p21

B. Activation of PI3-kinase

C. Addition of IGF to the medium

D. Stimulation of IGF receptors

203. Which of the following statements describes a differ-ence between the three mutant myoblast cell linesdescribed in the passage?

A. They fail to survive even in the presence of IGF.

B. They contain mutations in the myoblast differ-entiation pathway.

C. They are able to bind IGF in a normal manner.

D. They are blocked in the same step of IGF-medi-ated signal transduction pathway.

204. A developed vertebrate contains skeletal, smooth andcardiac muscle types, yet only skeletal muscle:

A. is striated.

B. is under voluntary control.

C. contains one nucleus per cell.

D. lines blood vessels.

Passage XI (Questions 205–208)

Apoptosis, or programmed cell death, is initiated by asignal instructing the cell to die. The process may beginwith the activation of a cell-surface receptor. This receptorcarries the death signal into the cell by modifying intracel-lular proteins, often by phosphorylation. Phosphorylationactivates the protein, which in turn modifies (and activates)another signaling protein. A series of proteins interact inthis manner to propagate the death signal to proteins insidethe mitochondria. Once the death signal reaches the mito-chondria, cytochrome C is released to activate caspases,calcium-dependent proteases that are always the finaleffectors of apoptosis. Caspases cleave important struc-tural proteins and other molecules.

Apoptosis is crucial for multiple functions of theimmune system. For example, some immature lympho-cytes are capable of targeting self antigens, while otherlymphocytes can recognize foreign antigens. Immaturelymphocytes recognizing self antigens are instructed to killthemselves through apoptosis. Millions of B-cell and T-cell precursors are destroyed in this manner. On the otherhand, lymphocytes recognizing foreign antigens are sparedthis death. In this way, the mature immune system createsonly lymphocytes capable of recognizing foreign antigens.

The cytotoxic lymphocyte is finely tuned to recognizeabnormal proteins such as those found in virally infectedand cancerous cells and induce apoptosis. This processstarts as the abnormal proteins are degraded into peptidesinside the altered cell and then bound to that cell’s MHCmolecules. This MHC-peptide complex is then displayedon the altered cell’s surface. If this complex is recognizedas foreign by a cytotoxic lymphocyte, the lymphocyteinstructs the altered cell to die. If the MHC displays nor-mal peptides, cytotoxic T-cells ignore the cell.

205. Which of the following would NOT result from adefect in a person’s apoptosis machinery?

A. Autoimmune disease

B. High cancer incidence

C. Lower than normal white blood cell count

D. Difficulty fighting viral infections

68

GO ON TO THE NEXT PAGE.

Page 70: Full length 7

206. Which of the following is a plausible method for acancerous cell to avoid a lethal immune response?

A. Cancerous cell shows normal MHC-peptidecomplexes, so it escapes detection by lympho-cytes.

B. Cancerous cell has effective apoptosis machin-ery, so it survives attack by cytotoxic lympho-cytes.

C. Cancerous cell invades the bloodstream to avoidlymphocytes, so it can easily survive forextended time periods.

D. Cancerous cell displays unusual MHC withviral peptides.

207. Which of the following, if blocked, would alwaysprevent apoptotic cell death in a variety of cell types?

A. Signaling of mitochondrial proteins

B. Antibody recognition

C. Caspases

D. CD8 receptors on T-cells

208. An impaired immune system, which results in theinability to kill virally infected cells but a normalability to produce antibodies, could be due to a defectin:

A. the bone marrow.

B. the thyroid gland.

C. B-cell maturation.

D. the thymus.

209. Which of the following compounds contains the leastbasic nitrogen atom?

210. At pH = 1.5, what is the net charge on glycine?

A. −1

B. 0

C. +1

D. +2

211. The (G+C):(A+T) ratio of a double stranded DNAmolecule from a certain strain of bacteria is found tobe 4:1. If this DNA molecule is replicated in a labo-ratory in a medium with a (G+C):(A+T) ratio of 1:1,what is the (G+C):(A+T) ratio of the daughter DNAstrand after one round of replication?

A. 1:1

B. 2:1

C. 4:1

D. 8:1

212. Which of the following is/are functions of the skin?

I. Physical barrier against pathogens

II. Regulation of body temperature

III. Excretion of excess water and salts

A. I only

B. I and II only

C. I and III only

D. I, II and III

A. NH3B. H2C=NHC. N

D.

H3CC

O

NH2

69

GO ON TO THE NEXT PAGE.

Questions 209 through 214 are NOT based on adescriptive passage.

Page 71: Full length 7

213. All of the following can only be found in the extra-cellular matrix (ECM) of eukaryotic cells EXCEPT:

A. collagen.

B. fibronectin.

C. proteoglycan.

D. myosin.

214. A man with blood type B whose mother was type Omarries a woman with blood type AB. Theoretically,what percentage of their children could donate bloodto a person of blood type B?

A. 0%

B. 25%

C. 50%

D. 100%

STOP. IF YOU FINISH BEFORE TIME IS CALLED,CHECK YOUR WORK.YOU MAY GO BACK TO ANYQUESTION IN THIS SECTION ONLY.

70